Интеллектуальные задачи с ответами: логические загадки и задачи онлайн

Содержание

Логические задачи для тренировки — Школа 52, Владивосток

Страница 1 из 2

Задачи для подготовки к самостоятельной работе.

 

1.  Министры иностранных дел России, США и Китая обсудили за закрытыми дверями проекты соглашения о полном разоружении, представленные каждой из стран. Отвечая затем на вопрос журналистов: «Чей именно проект был принят?», министры дали такие ответы:

Россия — «Проект не наш, проект не США»;
США — «Проект не России, проект Китая»;
Китай — «Проект не наш, проект России».
Один из них (самый откровенный) оба раза говорил правду; второй (самый скрытный) оба раза говорил неправду, третий (осторожный) один раз сказал правду, а другой раз — неправду.

Определите, представителями каких стран являются откровенный, скрытный и осторожный министры.

2.  В 8, 9 и 10-ом классах учатся три товарища: Наиль, Зиннур и Альберт. Один из них занимается в кружке информатики, другой -радиолюбитель, третий – авиамоделист. Скажите, в каком классе учится и в каком кружке занимается каждый из них, если известно, что:

8-й класс посетил лесхоз, 9-й – кондитерскую фабрику,
10-й класс — автозавод;
во время посещения лесхоза Зиннур опасался, как бы не разбить лежавшую в кармане радиолампу;
Наиль ушёл на стадион один, так как его товарищ был занят налаживанием своей модели самолёта;
4) товарищ авиамоделиста очень заинтересовался конвейером на автозаводе.

3. Однажды в международном лагере отдыха за круглым столом оказалось пятеро парней из Москвы, Санкт-Петербурга, Новгорода, Казани и Уфы. Их имена: Саша, Никита, Руслан, Петя и Миша.

Москвич сидел между уфимцем и Мишей, санкт-петербужец – между Сашей и Никитой, а напротив него сидели казанец и Руслан. Петя никогда не был в Санкт-Петербурге, а Саша не бывал в Москве и Уфе. Уфимец с Никитой регулярно переписываются.

В каком городе живёт каждый из ребят?

4. Лауреатом женского поэтического конкурса стали пять поэтесс.
1-е место заняла мисс Мартин. Бетти Мун не писала стихотворение «Сорока». Своё стихотворение Мэри сочинила в мае. «Муза» была написана в сентябре. Джастина Моран написала «Зрелый день». Стихотворение «Человек» принадлежит перу Кэт. «Муссон» был создан Сюзан в марте. Мисс Мегге написала своё стихотворение в апреле. Фамилия Мэри не Мульдон, а одно из стихотворений написано в октябре.

Назовите имя и фамилию каждой поэтессы, название стихотворения, которое она написала, а также время его создания.

5. Три свидетеля дорожного происшествия сообщили сведения о скрывшемся нарушителе. Боб утверждает, что тот был на синем «Рено». Джон сказал, что нарушитель ехал на черной «Тойоте», а Сэм сказал, что машина была точно не синяя, и, по всей видимости, это был «Форд». Когда удалось отыскать машину, выяснилось, что каждый из свидетелей точно определил только  один из параметров автомобиля, а в другом ошибся. Машина какой марки и какого цвета была у нарушителя?

6. Внимание Андрея, Дениса и Марата привлек промчавшийся мимо них автомобиль.
— Это английская машина марки «Феррари» — сказал Андрей.
— Нет, машина итальянская марки «Понтиак», — возразил Денис.
— Это «Сааб», и сделан он не в Англии, — сказал Марат.
Оказавшийся рядом знаток автомобилей сказал, что каждый из них прав только в одном из двух высказанных предположений.
Какой же марки этот автомобиль и в какой стране изготовлен?

7. Трое друзей, болельщиков автогонок «Формула-1», спорили о результате предстоящего этапа гонок.
— Вот увидишь, Шумахер не придет первым, — сказал Джон. — Первым будет Хилл.

— Да нет же, победителем будет, как всегда, Шумахер! — воскликнул Ник. — А об Алези и говорить нечего, ему не быть первым.
Питер, к которому обратился Ник, возмутился:
— Хиллу не видать первого места, а вот Алези пилотирует самую мощную машину.
По завершении этапа гонок оказалось, что каждое из двух предположений двоих друзей подтвердилось, а оба предположения третьего из друзей оказались неверны. Кто выиграл этап гонки?

8 Три дочери писательницы Дорис Кей — Джуди, Айрис и Линда тоже очень талантливы. Они приобрели известность в разных видах искусств — пении, балете и кино. Все они живут в разных городах, поэтому Дорис часто звонит им в Париж, Рим и Чикаго, Известно, что:

1) Джуди живет не в Париже, а Линда — не в Риме;
2) Парижанка не снимается в кино;
З) Та, кто живет в Риме, певица;
4) Линда равнодушна к балету.
Где живет Айрис и какова ее профессия?

9 В симфонический оркестр приняли на работу трех музыкантов — Брауна, Смита и Вессона, умеющих играть на скрипке, флейте, альте, кларнете, гобое и трубе. Известно, что:

1) Смит — самый высокий;
2) играющий на скрипке меньше ростом играющего на флейте;
З) играющие на скрипке и флейте и Браун любят пиццу;
4) когда между альтистом и трубачом возникает ссора, Смит мирит их;
5) Браун не умеет играть ни на трубе, ни на гобое.

На каких инструментах играет каждый из музыкантов, если каждый владеет двумя инструментами?

10. После традиционного вечера встречи с бывшими выпускника­ми школы в стенгазете появилась заметка о трех наших бывших учениках. В этой заметке было написано, что Иван, Андрей и Борис стали учителями. Теперь они преподают разные дисциплины: один — математику, второй — физику, а тре­тий — химию. Живут они тоже в разных городах: Минске, Витебске и Харькове. В заметке было еще написано, что первоначальные их планы осуществились не полностью: Иван работает не в Минске, Андрей — не в Витебске; житель Минска преподает не математику, Андрей преподает не физику. Повезло только жителю Витебска: он преподает любимую им химию.

Кто где живет и что преподает?

11. В педагогическом институте Казакова, Андреева, Покатина, Галеев, Шакиров и Веселов преподают философию, математику, английский язык, французский язык, немецкий язык, историю. 

Преподаватель немецкого языка и преподаватель математики в студенческие годы занимались художественной гимнастикой.
Шакиров старше Веселова, но стаж работы у него меньше, чем у преподавателя философии.
Будучи студентками, Казакова и Андреева учились вместе в одном университете. Все остальные окончили педагогический институт.

Веселов – отец преподавателя французского языка.
Преподаватель английского языка – самый старший из всех по возрасту и по стажу работы. Он работает в этом институте с тех пор, как окончил его. Преподаватели математики и истории – его бывшие студенты.
Казакова старше преподавателя немецкого языка.
Кто какой предмет преподаёт?

12. Одиннадцать ребят: Александр, Борис, Василий, Георгий, Дмитрий, Евгений, Захар, Иван, Кирилл, Леонид и Михаил – учатся все в разных классах одной школы.
Старший брат Дмитрия оканчивает 7-й класс, а младший брат Евгения учится в 5-ом классе. Александр старше Кирилла на один класс, Леонид старше Евгения на два класса, а самый старший из мальчиков Михаил. Борис помогает в учёбе Евгению, Дмитрий – Ивану, Георгий – Александру.

Иван при окончании 4-го класса получил похвальную грамоту.
3) Борис – вожатый в 5-ом классе, а Василий – в 4-ом классе.
4) Александр, Кирилл и шестиклассник занимаются в гимнастической секции, а одновременно с ними тренируются баскетболисты, среди которых всегда Борис, Евгений и восьмиклассник.
5) Александр и семиклассник живут на улице Лесной, Георгий и пятиклассник – на улице Красивой, Дмитрий, первоклассник и восьмиклассник – на Садовой, а Кирилл и десятиклассник – на Солнечной.
Кто из них в каком классе учится?

13. Три одноклассника — Влад, Тимур и Юра, встретились спустя 10 лет после окончания школы.
Выяснилось, что один из них стал врачом, другой физиком, а третий юристом.
Один полюбил туризм, другой бег, страсть третьего — регби.
Юра сказал, что на туризм ему не хватает времени, хотя его сестра — единственный врач в семье, заядлый турист.
Врач сказал, что он разделяет увлечение коллеги.
Забавно, но у двоих из друзей в названиях их профессий и увлечений не встречается ни одна буква их имен.
Определите, кто чем любит заниматься в свободное время и у кого какая профессия.

 

 

Логические задачи как часть собеседования QA-специалиста — TestMatick

Решаем логические задачи

Любое собеседование на должность инженера по тестированию программного обеспечения традиционно состоит из нескольких логически взаимосвязанных этапов.

Для начала, HR-специалист просит человека объяснить, почему именно эта профессия его заинтересовала и по каким именно причинам компания должна взять именно его (собеседника) на данную должность.

Затем технический специалист компании по найму сотрудников будет предлагать дать ответы на базовые вопросы по основам теории тестирования программного обеспечения. Так он моментально поймет, насколько хорошо человек владеет техническим и практическим материалом, а также как скоро он может приступить к изучению рабочей специфики непосредственно во время выполнения реальных заданий.

Затем наступает этап, с которым сталкиваются не все кандидаты. Речь идет о тестировании логического мышления человека. Можно задаться вопросом – а зачем это требуется? Все очень просто. На основе логических головоломок, HR-специалисты могут качественно проследить за ходом ваших мыслей и точно понять, есть ли у вас задатки аналитического ума или нет.

Тут важно не паниковать, не теряться, а стараться размышлять вслух, даже если вы совсем далеки от понимания того, как правильно решить ту или иную задачу.

Далее в статье речь пойдет о логических задачах, которые чаще всего встречаются при собеседовании в IT-фирмах. А также рассмотрим практические советы касательно того, как эффективно можно подготовиться к таким собеседованиям.

Топ логических задач

Логические задачи дают возможность техническим специалистам, проводящим собеседование, быстро формировать точное представление о том, какой кандидат перед ними находиться, какие технические задатки в нем присутствуют или могут быть раскрыты в будущем.

Головоломки и задачи не всегда предполагают под собой математическое решение. Порой задача может содержать сразу несколько однозначных ответов.

При собеседовании интервьюеры иногда могут прибегать к задачам для QA, которые часто используются во время найма сотрудников.

Далее как раз и рассмотрим несколько примеров таких задач, головоломок и логических построений, которые полезно взять на вооружение.

Логическая задача от Microsoft

Перед вами два мотка веревки. Если взять их концы и поджечь, то каждый моток сгорает за один час. Вопрос: как правильно отмерить 45 минут, применяя 2 таких мотка веревки, при условии, что веревку нет возможности делить?

Ответ: стоит одновременно поджечь один моток с двух концов, а второй – с одной стороны. Ровно через 30 минут первый моток полностью выгорит, а второму останется гореть 30 минут. Дабы получить желаемые 15 минут, его придется подпалить с обеих сторон. Все просто!

Интересная головоломка от Google

Допустим, перед вами 8 идентичных шариков. Как можно определить самый тяжелый из них, используя весы исключительно для двух взвешиваний?

Решение: берем 6 шаров, делим их на 2 группы и взвешиваем. Если одна из этих групп будет тяжелее, берем из нее 2 любых шара и снова их взвешиваем. Таким образом, вы либо сразу обнаружите шар тяжелее, либо им окажется третий шар из этой же группы. Если же самого тяжелого шарика ни в одной ни во второй группе не оказалось, значит он среди двух оставшихся.

Загадка от Adobe

Перед вами 50 мопедов с полным баком бензина на 100 километров. Как далеко можно уехать, при условии, что все они стартуют из одной точки?

Есть несколько решений:

  1. Заводим мопеды одновременно и едем 100 километров.
  2. Двигаем мопеды на 50 км, переливаем бензин из половины мопедов в другую половину. Так у нас получается 25 мопедов с полностью заполненным баком топлива. Едем дальше и повторяем данное действие каждые 50 км. Подобным образом можно продвинуться на 350 километров.

Вопрос о канализационном люке

Почему его форма круглая?

Здесь ответов может быть сразу несколько. Во-первых, равный диаметр круга люка не позволяет ему провалиться в колодец. Во-вторых, люки именно круглой формы перемещать намного проще, нежели квадратные.

К слову, подобный вопрос позволяет вам проявить на собеседовании фантазию и изобретательность.

Удивительное высказывание

Какая особенность содержится в следующем высказывании: «the quick brown fox jumps over the lazy dog»?

Ответ: данное высказывание содержит минимум по одной букве английского алфавита. Это задание в первую очередь направленно на вашу внимательность.

Как подготовить себя к подобным задачам?

Если вы хотите стать тестировщиком и в скором времени вам предстоит проходить собеседование, обязательно нужно подготовиться, отдельный акцент сделав на логические задачи.

Но как правильно это сделать?

  • Постоянная практика. Чем чаще вы решаете такие задачи, тем легче на собеседовании вам будет с ходу решить поставленную головоломку. Не нужно полагаться только на удачу. Тем более, что вы можете банально переволноваться в самый не подходящий момент;
  • Анализируйте каждую задачу. На просторах глобальной сети есть масса примеров задач с детальным разбором их содержания. Заучивать ничего не стоит, просто старайтесь разбирать задачу по пунктам и запоминайте ход выполнения.

В завершение

При прохождении собеседования на должность тестировщика ПО, HR-специалисты часто будут просить вас решить разные логические задачи.

Помните, что их цель не просто услышать единственно верное решение, а понаблюдать за ходом мышления, выявить его аналитические способности, а также желание искать решение нетривиальных задач самыми необычными способами.

‎App Store: LogicLike: игра в головоломки

10 увлекательных разделов с головоломками, которые хочется решать детям и взрослым:

— Текстовые и классические логические задачи: найди лишнее, соответствия, множества и многое другое.
— Загадки и вопросы на логику.
— Задания на пространственное мышление.
— Правда и ложь.
— Математические ребусы с буквами и цифрами, магические квадраты.
— Поиск закономерностей.
— Головоломки со спичками, перестановки.
— Алгоритмы.
— Комбинаторные задачи.
— Взвешивания и переливания.

— Шахматные задачи для начинающих.
— И другие занимательные головоломки.

В приложении Logiclike — вся логика от А до Я!

Вам больше не нужно быть «самому себе методистом», собирать информацию по крупицам и составлять собственный план занятий.
Перед вами уникальный систематизированный курс, развивающий учебно-игровой комплекс.
Над развитием платформы для вас работает целая команда педагогов, методистов, программистов и дизайнеров!

Коллекция логических задач и головоломок растет каждый месяц!
Всего в приложении 500+ логических задач, 400+ пространственных головоломок, 300+ ребусов и сотни других интересных заданий для развития смекалки.
Мы регулярно добавляем новые интересные задачи для ума.

Особенности и преимущества приложения

— Вся логика в вашем телефоне: 3500 заданий на любой вкус, 17 занимательных разделов.
— Безопасная среда для развития ребёнка — без рекламы и ссылок!
— Красочное оформление, современный дизайн, приятные цвета, от которых не устают глаза.
— Озвучка упражнений: начать занятия могут и дети, которые еще не умеют читать.
— Рейтинги учеников: поддерживают соревновательный дух и дополнительно мотивируют не сбавлять обороты.
— Звезды и награды за достижения.
— Сертификаты о прохождении отдельных этапов курса.
— Игровое интерактивное обучение.
— В сложной ситуации можно обратиться к советам Робота Клапана. Помощник Профессора натолкнет вас на правильный ход решения или направит в подходящий раздел теории.
— Подсказки Профессора рассказывают о новых и необычных подходах к решению задачи.
— Геймификация — заниматься легко и интересно.

Занимайтесь с любого устройства дома, в автомобильной пробке, на даче, после завтрака и в обеденный перерыв.

Вступайте в клуб логиков!
Мы очень стараемся, чтобы заниматься было интересно и очень полезно и детям, и взрослым. Установите приложение и попробуйте решить 10-15 задачек.

Во время игры развивайте и совершенствуйте важные навыки и способности:

1) Умение мыслить критически, нестандартно, вопреки шаблонам.
2) Умение размышлять и рассуждать логически правильно.
3) Умение доказывать истинность или ложность суждений, объяснять свою позицию себе и окружающим.
4) Интерес к новому, желание учиться и продуктивно работать с информацией.
5) Позитивное отношение к сложным задачам: воспитайте привычку воспринимать проблему как приключение.
6) Научитесь решать любые логические задачи и понимать, как находить ответы на самые сложные вопросы.
7) Будете принимать обоснованные решения, руководствуясь логикой, а не эмоциями или шаблонным мышлением.

Успешный опыт решения логических и нестандартных задач, повышает самооценку, уверенность в своих силах, вдохновляет на достижения в других сферах деятельности.

Современное приложение, которое объединяет!
Логика может стать вашим новым хобби.
Увлекательные логические задачи объединяют друзей, детей и родителей.
Решайте всей семьёй!

Соглашение: https://logiclike.com/docs/public.pdf?v=4

Тесты на логику и мышление

Воображение, творческие способности и абстрактное мышление повышают результативность работы на любой должности. Наличие этих качеств говорит о развитых когнитивных способностях (iq), эрудиции и обучаемости. Именно поэтому тесты на логику и мышление – обязательный этап при трудоустройстве во многие российские компании.

Подготовка к тестам на логику и мышление:      

Онлайн тесты 🔥Онлайн тесты 🔥

Где использую тесты на мышление и логику

Тесты на логику или на логическое мышление оценивают творческий склад ума у соискателя, воображение, умение визуализировать получаемую информацию.

Они анализируют склонности к пространственному и абстрактному мышлению, обобщению, расстановке приоритетов.

Онлайн-пример теста на логику и мышление:

Кроме того, по результатам этих испытаний HR оценивают:

  • скорость реакции;
  • умение переключаться;
  • уровень интеллекта, эрудиции;
  • склонность к критическому мышлению;
  • навыки анализа поступающей информации;
  • способности к визуализации и моделированию.

Способность абстрактно мыслить востребована не только среди представителей творческих профессий, но и среди управленцев и даже инженеров. Испытания на логику  используются для оценки и подбора:

  • дизайнеров, архитекторов;
  • маркетологов, PR-менеджеров;
  • креативных директоров;
  • инженеров, проектировщиков;
  • IT-специалистов, системных администраторов;
  • финансистов, бухгалтеров, экономистов;
  • госслужащих, сотрудников МВД, полиции;
  • менеджеров по продажам, закупкам, работе с клиентами.

Поскольку логические задачи и тесты предсказывают способности кандидатов с достаточно высокой точностью, их используют многие крупные компании, например:

Виды тестов на мышление

Логические задачи, используемые для оценки кандидатов при приеме на работу, отличаются от классических опросников в интернете на коэффициенты интеллекта. Это связано со спецификой тестов способностей.

Как правило, задания представляют собой набор геометрических фигур, цифр или знаков, объединенных некой закономерностью. Кандидату предстоит определить взаимосвязь и либо продолжить ряд фигур, либо найти среди них лишнюю.

Замечание:

Логические задания встречаются в комплексном тестировании SHL, talentQ, входят в состав КОТ и других тестов при приеме на работу.

Разработчики тестов на логику

SHL – одна из самых популярных консалтинговых компаний, предлагающая комплексную оценку кандидатов. Задания этого разработчика предсказывают успех кандидата на будущем рабочем месте, его потенциал, склонности, предрасположенности, таланты.

В тестах способностей SHL  используются геометрические фигуры, представленные в определенной последовательности. Кандидату придется понять логику и продолжить представленный ряд. Тестирование встречается в Пятерочке, Магните ВТБ, Сбербанке Тинькофф — банке.


КОТ — краткий оценочный  тест, определяющий общий уровень интеллекта, эрудиции, навыки восприятия числовой, вербальной и графической информации. Часть вопросов КОТ — логические задачи. В них требуется определить направление вращения, движения, изменения размера фигур и продолжить последовательность.

КОТ оценивает память, скорость мышления, навыки принятие решений. Логические элементы в тестировании оценивают пространственное воображение. Он используется при трудоустройстве в государственные структуры, МФД РФ, полицию.


Talent Q – компания-разработчик, создающая адаптивные тесты при приеме на работу. Их особенность – вопросы упрощаются или усложняются в зависимости от правильности предыдущих ответов.

В talent Q используется последовательность чисел, знаков и геометрических фигур, связанных между собой. В некоторых задачах придется найти лишний элемент, в других – продолжить последовательность. Задачи сложнее тех, что предлагает SHL.  Тестирование встречается при трудоустройстве в Яндекс, Сбербанк, Сколково, Роскосмос, Газпром.

Сложность при прохождении тестов на мышление

Даже кандидаты с опытом испытывают сложности при прохождении логических заданий. Это связано со спецификой подобного формата:

  1. Во-первых, абстрактное мышление редко используется в повседневной жизни.
  2. Во-вторых, задачи на определение последовательностей непросты – в одной фигуре до 7 элементов.

Задания на логику или на логическое мышление SHL и  talentQ используются в России недавно, поэтому российские соискатели еще не привыкли к подобному формату оценочных испытаний.

Все что нужно для прохождения оценочных испытаний – познакомиться с форматом задач и потренироваться определять связи и последовательности. Кроме того, желательно потренироваться в пространственном воображении – для этого также созданы соответствующие тренировочные упражнения, игры, руководства и онлайн тренажеры.

Важно помнить и об ограничении времени – как правило, на один вопрос дается не более 50 секунд, что еще сильнее усложняет прохождение тестирования. Поэтому в процессе подготовки важно уделять внимание не только подходу к задачам, но и концентрации, собранности и внимательности.

Особенности подготовки

Готовясь к собеседованию и тестам способностей, важно тренироваться последовательно, двигаясь от простого к сложному.

  1. Бессмысленно искать, где скачать бесплатное online тестирование на логику и мышление с ответами. У работодателей большой банк вопросов, и шансов, что двум кандидатам попадутся одни и те же вопросы, практически нет.
  2. Перед тем как приступать к подготовке, рекомендуется выяснить формат тестирования, который попадется на собеседовании. От этого зависит дальнейшая стратегия подготовки. В динамических испытаниях важно научиться определять сложные вопросы и вовремя пропускать их, чтобы успеть решить большее количество заданий.
  3. На первом этапе подготовки время не засекают. Важно понять принцип решения и выработать паттерны и тактику. После этого рекомендуется тренироваться с таймером.
  4. После каждой тренировочной сессии работают над ошибками. В каждом вопросе есть комментарии с правильным решением. Задания, вызвавшие трудности, проходят повторно.
  5. За день до собеседования рекомендуется сделать перерыв. Это поможет собрать силы и сконцентрироваться.

Заключение

Задания на логику оценивают скорость анализа информации, умение мыслить критически, и визуализировать данные. Тестирование предлагают на собеседованиях в магазины, банки, коммерческие и государственные структуры.

Хотя некоторые соискатели думают, что задачи не представляют сложности, на этом этапе отсеивается до 90% кандидатов. Поэтому к дистанционным испытаниям так важна подготовка, не говоря уже о подготовке к самому собеседованию.

Оцените статью

средняя оценка 5,00 (2 голосов)

Загрузка…

ЛОГИЧЕСКИЕ ЗАДАЧИ ДЛЯ ДОШКОЛЬНИКОВ Объясняйте и направляйте ребенка к правильному ответу — Блог — МБДОУ «ЦРР-детский сад №90»

Когда: 09 апреля 2020 г.

Где: СИДИМ ДОМА!!!

К сожалению  друзья! Мы сидим дома! Но это не мешает нам продолжать заниматься! Мы с Вами — в это непростое время!

Жираф, крокодил и бегемот
жили в разных домиках.
Жираф жил не в красном
и не в синем домике.
Крокодил жил не в красном
и не в оранжевом домике.
Догадайся, в каких домиках жили звери?

*****
Три рыбки плавали
в разных аквариумах.
Красная рыбка плавала не в круглом
и не в прямоугольном аквариуме.
Золотая рыбка — не в квадратном
и не в круглом.
В каком аквариуме плавала зеленая рыбка?

*****
Жили-были три девочки:
Таня, Лена и Даша.
Таня выше Лены, Лена выше Даши.
Кто из девочек самая высокая,
а кто самая низкая?
Кого из них как зовут?

*****
У Миши три тележки разного цвета:
Красная, желтая и синяя.
Еще у Миши три игрушки: неваляшка, пирамидка и юла.
В красной тележке он повезет не юлу и не пирамидку.
В желтой — не юлу и не неваляшку.
Что повезет Мишка в каждой из тележек?

*****
Мышка едет не в первом и не в последнем вагоне.
Цыпленок не в среднем и не в последнем вагоне.
В каких вагонах едут мышка и цыпленок?

*****
Стрекоза сидит не на цветке и не на листке.
Кузнечик сидит не на грибке и не на цветке.
Божья коровка сидит не на листке и не на грибке. Кто на чем сидит? (лучше все нарисовать)

*****
Алеша, Саша и Миша живут на разных этажах.
Алеша живет не на самом верхнем этаже и не на самом нижнем.
Саша живет не на среднем этаже и не на нижнем.
На каком этаже живет каждый из мальчиков?

*****
Ане, Юле и Оле мама купила ткани на платья.
Ане не зеленую и не красную.
Юле — не зеленую и не желтую.
Оле — не желтое и не красное.
Какая ткань для какой из девочек?

*****
В трех тарелках лежат разные фрукты.
Бананы лежат не в синей и не в оранжевой тарелке.
Апельсины не в синей и в розовой тарелке.
В какой тарелке лежат сливы?
А бананы и апельсины?

*****
Под елкой цветок не растет,
Под березой не растет грибок.
Что растет под елкой,
А что под березой?

*****
Антон и Денис решили поиграть.
Один с кубиками, а другой машинками.
Антон машинку не взял.
Чем играли Антон и Денис?

*****
Вика и Катя решили рисовать.
Одна девочка рисовала красками,
а другая карандашами.
Чем стала рисовать Катя?

*****
Рыжий и Черный клоуны выступали с мячом и шаром.
Рыжий клоун выступал не с мячиком,
А черный клоун выступал не с шариком.
С какими предметами выступали Рыжий и Черный клоуны?

*****
Лиза и Петя пошли в лес собирать грибы и ягоды.
Лиза грибы не собирала. Что собирал Петя?

*****
Две машины ехали по широкой и по узкой дорогам.
Грузовая машина ехала не по узкой дороге.
По какой дороге ехала легковая машина?
А грузовая?

 

10 математических и логических задач из собеседований в Apple, Google, Adobe и Microsoft

22 Февраля, 2015, 16:00

297021

Кому не хотелось бы устроиться на работу в Google, Intel, Amazon или Apple? Многие IT-компании славятся тем, что на собеседовании задают соискателям каверзные задачи на математику, логику и общую сообразительность. Наверное, один из самых знаменитых примеров — это вопрос о том, почему канализационные люки круглые. Редакция AIN.UA постаралась подобрать самые интересные примеры таких задач, для решения которых требуется знание математики на школьном уровне или просто смекалка. Некоторые из них приводят сами компании, некоторые — публикуют пользователи, которые ходили на собеседование, некоторые — собраны на популярных сайтах задач. Почти под каждой задачей приведен верный ответ (или, по крайней мере, один из возможных верных ответов), набранный шрифтом белого цвета — увидеть его можно, выделив соответствующую область.

Что спрашивают в Apple

1. Задача на логику. Шелдон Купер (тот самый гениальный физик из популярного сериала) дошел в игровом квесте в погоне за сокровищами до последнего рубежа. Перед ним — две двери, одна ведет к сокровищу, вторая — к смертельно опасному лабиринту. У каждой двери стоит стражник, каждый из них знает, какая дверь ведет к сокровищу. Один из стражников никогда не врет, другой — врет всегда. Шелдон не знает, кто из них врун, а кто нет. Прежде чем выбрать дверь, задать можно только один вопрос и только одному стражнику.

Вопрос: Что спросить Шелдону у стражника, чтобы попасть к сокровищу?

Ответ: Можно спросить любого, при этом задать вопрос так: «Какая дверь, по мнению другого стражника, правильная?». Если он спросит у правдивого, то получит данные о том, какая дверь ведет к лабиринту, ведь врущий стражник всегда врет. Если же он спросит у врущего стражника, опять же, узнает, какая дверь ведет к лабиринту, ведь тот соврет о двери, на которую укажет правдивый стражник. 

2. Землю захватили инопланетяне. Они планируют уничтожить всю планету, но решили дать человечеству шанс. Они выбрали десяток самых умных людей и поместили их в абсолютно темную комнату, посадив в ряд, один за другим. На каждого из людей надели по шляпе, шляпы всего двух цветов — розовые и зеленые. После того, как все шляпы оказываются на головах, свет включается.

Инопланетянин начинает с последнего человека в ряду и спрашивает о том, какого цвета шляпа у него на голове. Других слов, кроме цвета шляпы, произносить нельзя. Отмалчиваться — тоже. Если он отвечает верно, остается в живых, ошибается — его убивают.

Нельзя посмотреть, какого цвета ваша шляпа, но можно договориться о некоем принципе, по которому отвечать всем. Расположение шляп — случайное, комбинации могут быть любыми, вам видны все шляпы, которые расположены перед вами.

Вопрос: Что нужно отвечать, чтобы выжило как можно больше людей?

Ответ: Первый отвечающий считает количество зеленых шляп перед собой, если это нечетное число, он называет «зеленый», если четное — «розовый». Следующий, видя количество и цвет шляп перед собой, может таким образом вычислить, какого цвета шляпа у него на голове (к примеру, если зеленых все еще нечетное количество, то очевидно, что на нем — розовая), и так далее. Таким образом гарантированно выживают 9 из 10, а у первого отвечавшего шанс 1 к 1.  

Что спрашивают в Adobe

3. У вас 50 мотоциклов, с заполненным топливом баком, которого хватает на 100 км езды.

Вопрос: Используя эти 50 мотоциклов, как далеко вы сможете заехать (учитывая, что изначально они находятся в условно одной точке пространства)?

Ответ: Самый простой ответ: завести их все одновременно и проехать 100 км. Но есть и другое решение. Сначала переместите все мотоциклы на 50 км. Затем, перелейте топливо из половины мотоциклов в другую половину. У вас таким образом — 25 мотоциклов с полным баком. Проедьте еще 50 км и повторите процедуру. Так можно забраться на 350 км (не учитывая того топлива, которое останется от «лишнего» мотоцикла при разделе 25 надвое).

Что спрашивают в Microsoft

4. У вас бесконечный запас воды и два ведра — на 5 литров и 3 литра.

Вопрос: Как вы отмерите 4 литра?

Ответ: Наполните водой пятилитровое ведро и вылейте часть воды в трехлитровое. У вас сейчас 3 литра в маленьком ведре и 2 — в большом. Опустошите маленькое ведро и перелейте туда оставшиеся 2 литра из большого. Снова наполните большое ведро и перелейте из него воду в малое. Там уже есть 2 литра воды, так что долить придется литр, а в большом останется 4 литра.

5. У вас два отрезка веревки. Каждый таков, что если поджечь его с одного конца, он будет гореть ровно 60 минут.

Вопрос: Имея только коробку спичек, как отмерить с помощью двух отрезков такой веревки 45 минут (рвать веревки нельзя)?

Ответ: Один из отрезков поджигается с двух концов, одновременно с этим поджигается второй отрезок, но с одного конца. Когда первый отрезок догорит полностью, пройдет 30 минут, от первого также останется 30-минутный отрезок. Поджигая его с двух концов, получим 15 минут. 

Что спрашивают в Google

6. У вас имеется 8 шариков одинакового вида и размера.

Вопрос: Как найти более тяжелый шарик, используя весы и всего два взвешивания?

Ответ: Отберите 6 шариков, разделите их на группы по 3 шарика и положите на весы. Группа с более тяжелым шариком перетянет чашу. Выберите любые 2 шарика из этой тройки и взвесьте. Если тяжелый шарик среди них, вы это узнаете, если они весят одинаково — тяжелый тот, что остался. Если же более тяжелого шарика в группах по 3 шарика не оказалось, он — среди 2 оставшихся.

Что спрашивают в Qualcomm

7. Эту задачку описал пользователь, которого собеседовали на позицию senior systems ENGINEer. Он отметил в описании задачи, что у него был свой ответ, по поводу которого он долго спорил с человеком, проводившим собеседование.

Предположим, у нас происходит 10 пакетных передач данных по беспроводной сети. Канал не очень качественный, так что есть вероятность 1/10, что пакет данных не будет передан. Трансмиттер всегда знает, удачно или неудачно был передан пакет данных. Когда передача неудачная, трансмиттер будет передавать пакет до тех пор, пока не преуспеет.

Вопрос: Какую пропускную способность канала получаем?

Ответ: По версии пользователя, ответ должен был быть 9 пакетов в секунду. Но человек, проводивший интервью, с ним не согласился, правда, ответа не назвал, но повторял, что «из-за ретрансмиссии пропускная способность должна быть уменьшена больше, чем на 1/10». 

Что спрашивают в «Яндексе»

8. Эту задачу предлагали решить для вступления в Школу анализа данных в феврале 2014 года. Ответа на задачи из «Яндекса» у нас, к сожалению, нет.

Игра состоит из одинаковых и независимых конов, в каждом из которых выигрыш происходит с вероятностью p. Когда игрок выигрывает, он получает 1 доллар, а когда проигрывает — платит 1 доллар. Как только его капитал достигает величины N долларов, он объявляется победителем и
удаляется из казино.

Вопрос: Найдите вероятность того, что игрок рано или поздно проиграет все деньги, в зависимости от его стартового капитала K.

9. Эту задачу предлагали решить разработчикам на собеседовании, и она больше связана непосредственно с программированием, чем предыдущие примеры.

Имеется морфологический словарь объемом примерно 100 000 входов, в котором глаголы совершенного и несовершенного вида помещены в отдельные статьи (то есть «делать» и «сделать» считаются разными словарными входами). Вам требуется найти в словаре такие видовые пары и «склеить» статьи в одну.

Вопрос: Опишите общий сценарий решения такой задачи и примерный алгоритм поиска видовых пар.

И бонус

10. Эту задачу приписывают Альберту Эйнштейну — якобы с ее помощью он подбирал себе ассистентов. Другая почти легендарная история приписывает авторство Льюису Кероллу. Отметим, что она очень просто решается на бумаге, но если хотите хардкора — попробуйте решить в уме.

  1. На улице стоят пять домов.
  2. Англичанин живет в красном доме.
  3. У испанца есть собака.
  4. В зеленом доме пьют кофе.
  5. Украинец пьет чай.
  6. Зеленый дом стоит сразу справа от белого дома.
  7. Тот, кто курит Old Gold, разводит улиток.
  8. В желтом доме курят Kool.
  9. В центральном доме пьют молоко.
  10. Норвежец живет в первом доме.
  11. Сосед того, кто курит Chesterfield, держит лису.
  12. В доме по соседству с тем, в котором держат лошадь, курят Kool.
  13. Тот, кто курит Lucky Strike, пьет апельсиновый сок.
  14. Японец курит Parliament.
  15. Норвежец живет рядом с синим домом.
  16. Каждый из домов покрашен в отдельный цвет, в каждом доме живет представитель отдельной национальности, у каждого — свой питомец, своя любимая марка сигарет и напиток.

Вопрос: Кто пьет воду? Кто держит зебру?

Ответ: Японец держит зебру, норвежец пьет воду. 

Задачи по математике для дошкольников

Дата: 4 марта 2014 Автор: Алина Рубрика: Для дошкольников

Логические задачи по математике для дошкольников – это не простые задачи. Потому что главное здесь не найти правильный ответ (их может быть несколько или не быть вовсе), а научиться мыслить, сравнивать, анализировать, делать выводы, пользоваться своим жизненным опытом и наблюдательностью. А также логические задачи – это отличный способ развить то, что называют смекалкой. Поэтому они такие интересные и совсем не скучные. А если отнестись к ним серьёзно, но легко и с юмором, то можно найти по-настоящему креативные решения. Попробуем?

Задачки на смекалку

1. У меня есть 3 домашних питомца, а всего у них 10 лап. Как такое может быть?
(Один из питомцев пернатый – у него 2 лапки, а двое других четвероногие. Но возможны и другие варианты – например, питомцами бывают и экзотические насекомые, у них 6 лапок, а есть и вовсе безлапые питомцы.)

2. Во дворе растут 3 дерева – берёза, клён и рябина. Грач сел не на берёзу, синица не захотела садиться на дерево без вкусных ягод. Куда сел воробей?
(Воробей сел на берёзу, грач на клён, а синица на рябину)

3. В пенале у Аси лежат 4 простых карандаша и 2 цветных. Какое минимальное количество предметов нужно взять, чтобы в руке точно оказался простой карандаш? (3 предмета)

4. Алёша пилил дрова. Сколько ему нужно сделать распилов, чтобы разделить бревно на 2 части? А на 3 части? А если бы он не дрова пилил, а делил бублик, сколько нужно сделать «распилов», чтобы отрезать от бублика кусочек?

5. Лена, Аня и Женя помогали бабушке в саду. Бабушка захотела сделать детям подарки. «Куплю-ка я куклы обеим девочкам», – подумала бабушка. Неужели она ошиблась и неправильно сосчитала детей?
(Бабушка не ошиблась, девочек было действительно двое: Лена и Аня, а мальчику Жене бабушка купила гоночную машинку)

6. Сыщик в панике: он обнаружил возле дома странные следы. Сначала от дома вели следы взрослого, маленького ребёнка и птицы (видимо, вороны). Ровно через 10 шагов следы птицы обрываются. Ещё через 10 шагов обрываются следы ребёнка. Это похищение! Прав ли сыщик?
(Скорее всего, сыщик не прав. Птица просто взлетела, а ребёнка взял на руки взрослый. Впрочем, вариант, когда ребёнок попятился домой по своим собственным следам, тоже не лишён некоторой логики. )

Задачка на “засыпку”

8. Черепаха пришла в гости к своей подруге, которая живёт на 14 этаже. Лифт оказался сломан – придётся подниматься пешком. Черепаха начала свой путь после полудня. Полчетвёртого уже – черепаха на четвёртом этаже. Полпятого уже – черепаха на пятом этаже. Полвосьмого уже – черепаха на восьмом этаже. Когда же она доберётся до 14 этажа?
(Полвторого ночи)

Задачи на логику для самых маленьких

Игра «Что лишнее?», как говорится, классика жанра. Но мир вокруг многовариантен, и не всегда ответы на жизненные вопросы однозначны. Предложим ребятам задачи с несколькими решениями. Если ребёнок найдёт сам все из них – замечательно. Если увидит только одно – не страшно, главное, чтобы он мог объяснить свой выбор, а другой вариант решения можно найти вместе.

1. Найди лишнее:

Коза, лось, кабан (коза, потому что это домашнее животное, или лось, потому что слово начинается на букву «Л», или кабан, потому что у него нет рогов)
Тыква, персик, перец (тыква, так как слово начинается на «Т», персик – он растёт в саду, а не в огороде, как остальные, или перец – у него форма совсем не шарообразная, а у тыквы и персика всё же более круглая.
Ложка, скалка, совочек (совочек, потому что это не кухонная утварь, скалка – ложка и совочек похожи по форме и по значению, а скалка отличается, или ложка – отличается первой буквой).

2. Не верь своим глазам!

Нарисуем 3 домика и пронумеруем их. На каждом домике висит табличка, мы знаем точно: то, что написано на ней – неправда!
Табличка на домике №1: «Тут живёт Пятачок»
Табличка на домике №2: «Кролик в третьем домике»
Табличка на домике №3: «Винни-Пуха здесь нет!»
В каком домике живёт каждый из героев?
(1 – Кролик, 2 – Пятачок, 3 – Винни-Пух)

3. На листе бумаги напишем вразнобой цифры от 1 до 20. Задача ребёнка соединить их подряд таким образом, чтобы линии не пересекались между собой.

Автор задач: Юлия Белка

Оцените статью: Поделитесь с друзьями!

Метки:

Примеры вопросов IQ с ответами

Термин «IQ» — это сокращение от термина «коэффициент интеллекта». IQ определяется путем проведения оценок, предназначенных для определения уровня интеллекта человека. Этот уровень интеллекта выражается отношением умственного возраста к хронологическому возрасту. Тесты на IQ часто используются для определения интеллекта соискателей вакансий, военных, студентов и других. Тест на IQ обычно проводится психологом; но в Интернете можно найти множество неформальных оценок IQ.

Вопросы по тесту IQ

Вопросы по тесту IQ можно найти в тестах IQ. Эти вопросы предназначены для оценки различных умственных способностей и навыков и, следовательно, охватывают широкий спектр различных типов интеллекта. Ниже приведены некоторые общие примеры типов вопросов, которые могут быть найдены в тесте IQ:

  • Аналогии (математические и словесные)
  • Управляемые закономерности (пространственные и математические)
  • Классификация
  • Визуальный
  • Пространственный
  • Логический

Хотя это общие области, которые можно изучить с помощью теста IQ, полезно увидеть более конкретные вопросы.Вот несколько тестовых вопросов, с которыми можно встретиться при прохождении теста IQ:

  • Какое число должно быть следующим в шаблоне?

37, 34, 31, 28

Ответ: 25, числа уменьшаются на 3

  • Найдите ответ, который лучше всего завершает аналогию:

Книга предназначена для чтения как вилка:

а. чертеж

б. письменная

с. перемешивание

д. ест

Ответ: d.

  • Найдите два слова, по одному из каждой группы, наиболее близкие по значению:

Группа A

разговорчивый, работа, восторженный

Группа B

злой, ветер, болтливый

a.разговорчивый и ветреный

б. работа и злой

c. разговорчивый и разговорчивый

д. экстатический и сердитый

Ответ: ок. Разговорчивый и разговорчивый

  • Что из следующего можно объединить в 5-буквенное английское слово?

а. H R G S T

б. Р И Л С А

с. Т О М Т

г. W Q R G S

Ответ: б. рельсы и c. девиз

  • Какое число лучше всего завершает аналогию:

8: 4 как 10:

a.3

б. 7

c.24

d.5

Ответ 5, потому что 4 — половина от 8, а 5 — половина от 10.

Могут встречаться и другие типы вопросов. Эти вопросы могут быть любыми из следующих:

  • Вопросы, которые показывают изображения игральных костей и спрашивают, какая из них отличается.
  • Вопросы с изображениями фигур в разных положениях и спрашивали, какая из них не принадлежит.
  • Вопросы с изображениями кубиков Рубика с просьбой правильно упорядочить изображения.
  • Вопросы с изображениями развернутых кубов с рисунками на каждой грани куба, спрашивающие, какой из вариантов выбора является изображением, которым был бы развернутый куб, если бы он был сложен.
  • Вопросы с изображениями развернутых форм, таких как прямоугольные призмы и пирамиды, с просьбой к испытуемому определить, в какой трехмерной форме развернутая бумага будет после сложения.
  • Таблицы, в которых все поля заполнены числами, а тестируемого просят указать, какой номер находится в пропущенном бланке.
  • Изображения перекрывающихся форм и тестируемый должен определить, какая диаграмма не принадлежит.
  • Вопросы, требующие не только расшифровки слова, но и определения категории, к которой оно принадлежит.
  • Вопросы, требующие идентификации недостающих частей.
  • Изображение ряда фигур, требующее от испытуемого определить, какая из них будет следующей.

Количество вопросов, на которые вы ответили правильно, затем используется для оценки теста и оценки вашего интеллекта.

Критическое мышление: основные вопросы и ответы

Хорошо думать — значит налагать дисциплину и сдержанность на наше мышление с помощью интеллектуальных стандартов, чтобы поднять наше мышление до уровня «совершенства» или качества. неестественно или вероятно в недисциплинированных, спонтанных мыслях. Наименее изученным аспектом критического мышления являются «интеллектуальные стандарты». Большинство учителей не учились оценивать мышление с помощью стандартов; действительно, часто мышление самих учителей очень «недисциплинировано» и отражает отсутствие внутренних интеллектуальных стандартов.

Вопрос: Не могли бы вы привести пример?

Пол: Безусловно, одно из самых важных различий, которое учителя должны регулярно проводить и которое требует дисциплинированного мышления, — это различие между рассуждением и субъективной реакцией.

Если мы пытаемся развить качественное мышление, мы не хотим, чтобы студенты просто отстаивали вещи; мы хотим, чтобы они пытались рассуждать на основе доказательств и веских причин.Часто учителя не понимают этого основного различия. Многие учителя склонны воспринимать беглую и остроумную или бойкую и забавную письменную или забавную речь учащихся за хорошее мышление. Они часто не имеют четкого представления о составляющих хорошей аргументации. Следовательно, даже если ученик может просто утверждать что-то, а не рассуждать вообще, если он делает это с живостью и яркостью, учителя склонны считать это равносильным хорошему рассуждению.

Это было ясно продемонстрировано в недавней письменной оценке штата Калифорния, в которой учителя и тестировщики аплодировали студенческому эссе, которое, по их словам, иллюстрировало «исключительные достижения» в обоснованной оценке, эссе, в котором вообще не было никаких рассуждений, это было не более чем чем одна субъективная реакция за другой. (См. « Почему ученики и учителя не рассуждают правильно, »)

Оценивающие учителя и тестировщики не заметили, что ученик не ответил на указания, не подтвердили свое суждение причинами и доказательствами, не рассматривал возможные критерии, на которых основывалось его суждение, не анализировал предмет в свете критериев и не выбирал доказательства, которые явно подтверждали его суждение. Вместо этого ученик:

описал эмоциональный обмен

утверждал — без доказательств — некоторые сомнительные утверждения

выражал различные субъективные предпочтения

Оценивающие учителя, по-видимому, недостаточно ясно понимали природу оценочного мышления или основные понятия о критериях, доказательствах, причинах и хорошо обоснованном суждении, позволяющем заметить несоответствие.В результате, кстати, было продемонстрировано явно неправильно оцененное студенческое эссе по всей стране (в ASCD Developing Minds ), систематически вводя в заблуждение около 150 000 учителей, которые читали эту публикацию.

Вопрос: Может ли это быть редкой ошибкой, не отражающей знания учителя?

Пол: Я так не думаю. Позвольте мне предложить способ, с помощью которого вы можете начать проверку моего утверждения. Если вы знакомы с какими-либо программами развития навыков мышления, спросите кого-нибудь, кто знает об этом, «Где говядина? вопрос.А именно: «Какие интеллектуальные стандарты сформулированы и преподаются в программе?» Я думаю, вы сначала обнаружите, что этот человек озадачен тем, что вы имеете в виду. И затем, когда вы объясните, что вы имеете в виду, я думаю, вы обнаружите, что этот человек не может сформулировать такие стандарты. Программы развития навыков мышления без интеллектуальных стандартов созданы для неправильного обучения. Например, одна из основных программ просит учителей побуждать учеников делать выводы и использовать аналогии, но ничего не говорит о том, как научить учеников оценивать сделанные ими выводы, а также сильные и слабые стороны используемых аналогий. Это упускает из виду суть. Идея состоит не в том, чтобы помочь учащимся делать больше выводов, а в том, чтобы делать обоснованные, не в том, чтобы помочь учащимся найти больше аналогий, а сделать более полезные и проницательные.

Вопрос: Как решить эту проблему? Как с практической точки зрения решить эту проблему?

Пол: Ну, без дополнительных уловок или быстрых исправлений. Не с большим пухом для учителей. Только при качественном долгосрочном развитии персонала, который помогает учителям в течение длительного периода времени, в течение многих лет, а не месяцев, работать над своим собственным мышлением и примириться с тем, что такое интеллектуальные стандарты, почему они важны и как преподавать для них.Государственный департамент на Гавайях имеет именно такую ​​долгосрочную, качественную программу критического мышления (см. «Программу наставничества»). Так что это одна из моделей, на которую могут взглянуть ваши читатели. Кроме того, Национальный совет по обучению критическому мышлению сосредоточен именно на формулировании стандартов мышления. Я надеюсь, что в конечном итоге, благодаря таким усилиям, мы сможем перейти от поверхностного к содержательному в развитии качественного мышления учащихся. Нынешний уровень обучения мышлению действительно очень низкий.

Вопрос: Но есть много областей, вызывающих беспокойство в обучении, не только одно, не только критическое мышление, но и коммуникативные навыки, решение проблем, творческое мышление, совместное обучение, самооценка и так далее. Как районы справляются со всем спектром потребностей? Как они могут сделать все это, а не просто одно, каким бы важным оно ни было?

Пол: Это ключ. Все необходимое для образования поддерживает все остальное, необходимое для образования.Только когда хорошие вещи в образовании рассматриваются поверхностно и ошибочно, они кажутся разрозненными, связкой отдельных целей, скоплением отдельных проблем, как множество пчел в мешке. Фактически, любая хорошо продуманная программа критического мышления требует интеграции всех навыков и способностей, которые вы упомянули выше. Следовательно, критическое мышление — это не набор навыков, который нельзя отделить от совершенства в общении, решении проблем, творческого мышления или совместного обучения, а также он не безразличен к чувству собственного достоинства.

Вопрос: Не могли бы вы вкратце объяснить, почему это так?

Пол: Сначала подумайте о критическом мышлении. Мы мыслим критически, когда нам нужно решить хотя бы одну проблему. Следовательно, человек не обладает хорошим критическим мышлением, если не решает никаких проблем. Если нет проблем, нет смысла мыслить критически. Обратное тоже верно. Решение некритических проблем непонятно. Невозможно эффективно решать проблемы, если не думать критически о природе проблем и о том, как их решать.Таким образом, обдумывание пути решения проблемы — это критическое мышление, а не что-то еще. Более того, критическое мышление, поскольку оно включает в себя нашу новую разработку собственного мышления по предмету, и поскольку наше собственное мышление всегда является уникальным продуктом нашего самоструктурированного опыта, идей и рассуждений, по сути является новым «творением», новое «создание», новый набор каких-то когнитивных и аффективных структур. Короче говоря, всякое мышление является результатом работы ума, и когда оно дисциплинировано для того, чтобы хорошо интегрироваться в наш опыт, оно является новым творением именно из-за неизбежной новизны этой интеграции.И когда это помогает нам решать проблемы, которые мы не могли решить раньше, это правильно называется «творческим».

«Создание» и «испытание этого создания» тесно взаимосвязаны. При критическом мышлении мы создаем и формируем идеи и опыт, чтобы их можно было использовать для структурирования и решения проблем, формирования решений и, в зависимости от обстоятельств, эффективного общения с другими. Создание, формирование, тестирование, структурирование, решение и общение — это не разные действия фрагментированного разума, а одно и то же целостное целое, рассматриваемое с разных точек зрения.

Вопрос: Как здесь сочетаются коммуникативные навыки?

Пол: Некоторое общение — это поверхностное общение, банальное общение — поверхностное и тривиальное общение на самом деле не требует образования. Все мы можем вести светскую беседу, можем делиться сплетнями. И нам не требуются какие-либо сложные навыки, чтобы делать это достаточно хорошо. Когда общение становится частью нашей образовательной цели, так это в чтении, письме, устной речи и аудировании. Это четыре способа общения, которые необходимы для обучения, и каждый из них представляет собой способ рассуждения.Каждая из них связана с проблемами. Каждый из них пронизан потребностями критического мышления. Возьмем, казалось бы, простой вопрос чтения книги, которую стоит прочитать. В книге автор развила свое мышление, взяла некоторые идеи и в некотором роде представила их в развернутой форме. Наша работа как читателя — переводить значение автора в значения, которые мы можем понять.

Это сложный процесс, требующий критического мышления на каждом этапе.

Для чего предназначена книга?

Чего пытается добиться автор?

Какие вопросы или проблемы возникают?

Какие данные, какой опыт, какие доказательства приводятся?

Какие концепции используются для организации этих данных, этого опыта?

Как автор думает о мире?

Является ли ее мышление оправданным, насколько мы можем видеть с нашей точки зрения?

А как она это оправдывает со своей точки зрения?

Как мы можем войти в ее точку зрения, чтобы оценить то, что она говорит?

Все эти вопросы возникают у критически настроенного читателя. И критический читатель в этом смысле — это просто тот, кто пытается примириться с текстом.

Итак, если кто-то некритически читает, писатель, оратор или слушатель, он вообще не является хорошим читателем, писателем, оратором или слушателем. Чтобы делать что-либо из этого хорошо, нужно при этом мыслить критически и в то же время решать конкретные проблемы коммуникации, а значит, эффективно общаться.

Короче говоря, коммуникация — это всегда транзакция между как минимум двумя логиками.В чтении, как я уже сказал, есть логика мышления автора и логика мышления читателя. Критический читатель реконструирует (и таким образом переводит) логику писателя в логику мышления и опыта читателя. Это влечет за собой дисциплинированную интеллектуальную работу. Конечный результат — новое творение; мышление писателя впервые существует в сознании читателя. Неплохой подвиг!

Вопрос: А самооценка? Как это вписывается?

Paul: Здоровая самооценка возникает из обоснованного чувства собственного достоинства, так же как самооценка возникает из компетентности, способностей и подлинного успеха. Если человек просто чувствует себя хорошо без уважительной причины, то он либо высокомерен (что, конечно, нежелательно), либо, наоборот, испытывает опасное чувство неуместной уверенности. Подростки, например, иногда настолько хорошо думают о себе, что действуют иллюзией, что могут безопасно водить машину в нетрезвом виде или безопасно принимать наркотики. Они часто слишком высоко ценят свою компетентность и способности и слишком не осознают свои ограничения. Чтобы точно отделить подлинную самооценку от ложного чувства собственного достоинства, требуется, как вы уже догадались, критическое мышление.

Вопрос: И, наконец, как насчет совместного обучения? Как это вписывается?

Paul: Совместное обучение желательно, только если оно основано на дисциплинированном критическом мышлении. Без критического мышления совместное обучение может превратиться в совместное неправильное обучение. Это коллективное плохое мышление, при котором плохое мышление, которое разделяют, подтверждается. Помните, сплетни — это форма совместного обучения; идеологическая обработка в группах сверстников — это форма совместного обучения; массовая истерия — это форма быстрого совместного обучения (массовое обучение самого нежелательного типа).Мы вместе изучаем предрассудки, социальную ненависть и страхи вместе, стереотипы и ограниченность ума — вместе. Если мы не вкладываем дисциплинированное критическое мышление в сердце и душу сотрудничества, мы получаем способ сотрудничества, который противоречит образованию, знаниям и пониманию.

Итак, есть много важных образовательных целей, глубоко связанных с критическим мышлением, так же как критическое мышление глубоко связано с ними. В основном проблема в школах заключается в том, что мы разделяем вещи, относимся к ним изолированно и в результате плохо с ними обращаемся.Таким образом, мы получаем поверхностное представление о каждой отдельной вещи, которая имеет важное значение для образования, вместо того, чтобы видеть, как каждая важная хорошая вещь помогает информировать все остальные

Вопрос: Одной из важных целей обучения должно быть создание климат, который пробуждает у детей чувство удивления и вдохновляет их воображение. Что могут сделать учителя, чтобы «зажечь» эту искру и сохранить ее в образовании?

Павел: Прежде всего, мы убиваем детское любопытство, ее желание глубоко задавать вопросы, поверхностными дидактическими инструкциями.Маленькие дети постоянно спрашивают, почему. Почему это и почему? И почему это другое? Но вскоре мы подавляем это любопытство бойкими ответами, ответами, чтобы отбиваться, а не отвечать на логику вопроса. В каждой области знаний каждый ответ порождает больше вопросов, поэтому чем больше мы знаем, тем больше мы осознаем, что не знаем. Только люди, у которых мало знаний, считают свои знания полными и целостными. Если бы мы глубоко задумались почти над любым из ответов, которые мы бойко даем детям, мы бы осознали, что на самом деле у нас нет удовлетворительного ответа на большинство их вопросов.Многие из наших ответов — не более чем повторение того, что мы в детстве слышали от взрослых. Мы передаем неправильные представления наших родителей и их родителей. Мы говорим то, что слышали, а не то, что знаем. Мы редко присоединяемся к квесту с нашими детьми. Мы редко признаем свое невежество, даже самому себе. Почему с неба идет дождь? Почему снег холодный? Что такое электричество и как оно проходит по проводам? Почему люди плохие? Почему существует зло? Почему идет война? Почему моя собака должна была умереть? Почему распускаются цветы? Действительно ли у нас есть хорошие ответы на эти вопросы?

Вопрос: Как любопытство сочетается с критическим мышлением?

Пол: Для процветания любопытство должно перерасти в дисциплинированное исследование и размышление.Предоставленный самому себе, он взлетит как воздушный змей без хвоста, то есть прямо в землю! Интеллектуальное любопытство — важная черта ума, но для ее реализации требуется целый ряд других черт. Это требует интеллектуального смирения, интеллектуальной смелости, интеллектуальной целостности, интеллектуальной настойчивости и веры в разум. В конце концов, интеллектуальное любопытство не вещь само по себе — ценно само по себе и для себя. Это ценно, потому что может привести к знанию, пониманию и пониманию; потому что это может помочь расширить, углубить, отточить наш разум, делая нас лучше, гуманнее и более одаренных людей.

Чтобы достичь этих целей, разум должен быть более чем любопытным, он должен быть готов работать, готов страдать от замешательства и разочарования, готов столкнуться с ограничениями и преодолевать препятствия, открыт для взглядов других и готов принимать идеи что многие люди считают опасным. То есть нет смысла пытаться моделировать и поощрять любопытство, если мы не желаем создавать среду, в которой умы наших учеников могут познать ценность и боль тяжелого интеллектуального труда.Мы оказываем нашим ученикам медвежью услугу, если намекаем, что все, что нам нужно, — это безудержное любопытство, что только с его помощью знания приходят к нам с блаженной легкостью в атмосфере веселья, веселья и веселья.

Что толку в любопытстве, если мы не знаем, что делать дальше и как его удовлетворить? Мы можем создать среду, необходимую для дисциплины, силы, радости и работы критического мышления, только моделируя ее до наших учеников и вместе с ними. Они должны видеть, как работает наш разум. Наш разум должен стимулировать их вопросы и еще один вопрос; вопросы, проверяющие информацию и опыт; вопросы, требующие объяснения причин и доказательств; вопросы, которые побуждают студентов исследовать интерпретации и выводы, исходя из фактов и опыта; вопросы, которые помогают учащимся раскрыть свои предположения, вопросы, которые побуждают учащихся следить за последствиями своей мысли, проверять свои идеи, разбирать свои идеи, подвергать сомнению свои идеи, серьезно относиться к своим идеям.Именно в этой атмосфере строгого интеллекта и процветает естественное любопытство.

Вопрос: Для наших студентов важно быть продуктивными членами рабочей силы. Как школы могут лучше подготовить учеников к решению этих проблем?

Пол: Фундаментальная характеристика мира, в который сейчас входят студенты, — это постоянно ускоряющиеся изменения; мир, в котором информация умножается, хотя она быстро устаревает и устаревает; мир, в котором идеи постоянно реструктурируются, проверяются и переосмысляются; где невозможно выжить, имея только один образ мышления; где нужно постоянно адаптировать свое мышление к мышлению других; где нужно уважать потребность в точности, аккуратности и скрупулезности; мир, в котором профессиональные навыки должны постоянно улучшаться и совершенствоваться — даже трансформироваться. Нам никогда раньше не приходилось сталкиваться с таким миром. Образованию никогда раньше не приходилось готовить учащихся к такому динамическому потоку, непредсказуемости и сложности для такого брожения, смятения и беспорядка.

Сейчас мы, преподаватели, находимся на линии огня.

Готовы ли мы коренным образом пересмотреть наши методы обучения?

Готовы ли мы к 21 веку?

Готовы ли мы изучать новые концепции и идеи?

Готовы ли мы научиться новому чувству дисциплины, обучая этому наших студентов?

Готовы ли мы привнести новую строгость в собственное мышление, чтобы помочь нашим ученикам привнести такую ​​же строгость в свои?

Готовы ли мы, короче говоря, стать критически настроенными мыслителями, чтобы быть примером того, что наши ученики должны усвоить и стать?

Это серьезные проблемы для профессии.Они призывают нас делать то, к чему не призывали ни одно предыдущее поколение учителей. Тем из нас, кто готов заплатить цену, все же придется учить бок о бок с учителями, не желающими платить цену. Это сделает нашу работу еще более сложной, но не менее увлекательной, не менее важной и не менее полезной. Критическое мышление — это суть хорошо продуманной реформы и реструктуризации образования, потому что оно лежит в основе изменений 21 века. Будем надеяться, что у многих из нас хватит силы духа и дальновидности, чтобы осознать эту реальность и соответствующим образом изменить нашу жизнь и наши школы.

Вопрос: Национальные стандарты приведут к национальной отчетности. Какое у тебя видение будущего?

Пол: Большая часть проведенных нами до сих пор национальных оценок основана на обучении и мышлении низшего порядка. Он сосредоточен на том, что можно назвать поверхностным знанием. Он вознаграждает способ мышления, допускающий множественный выбор машинной оценки. Теперь мы признаем, что оценка будущего должна быть сосредоточена на мышлении высшего, а не низшего порядка; что он должен оценивать больше рассуждений, чем вспоминать; что он должен оценивать подлинные выступления студентов, занимающихся добросовестным интеллектуальным трудом.

Наша задача заключается в разработке и проведении такой оценки. В ноябре прошлого года мы с Джеральдом Носичем разработали и представили по запросу Министерства образования США модель национальной оценки мышления высшего порядка. На последующем собрании ученых и практиков, занимающихся решением проблем, общением и тестированием критического мышления, было почти единогласно решено, что можно оценить мышление более высокого порядка в национальном масштабе.Из обязательств департаментов образования, труда и торговли было ясно, что такая оценка уже заложена.

Дело в том, что у нас должны быть стандарты и стратегии оценки для мышления высшего порядка по ряду причин.

Во-первых, оценка и подотчетность никуда не денутся. Общественность меньшего не примет.

Во-вторых, то, что не оценивается, в целом не преподается.

В-третьих, то, что неправильно оценивают, неправильно учат.

В-четвертых, мышление высшего порядка, способности критического мышления становятся все более важными для успеха во всех сферах личной и профессиональной жизни.

В-пятых, исследования критического мышления делают возможным развитие и оценку мышления высшего порядка.

Путь будет нелегким, но если мы возьмем знания, понимание и понимание, которые мы приобрели в области критического мышления за последние двенадцать лет, мы можем многое сделать в оценке, чего мы еще не сделали. — на уровне отдельного классного руководителя, на уровне школьной системы, на уровне государства и на уровне страны.

Конечно, мы хотим сделать это таким образом, чтобы не допустить «Гарвардского заблуждения»; ошибочное представление о том, что, поскольку выпускники Гарварда очень успешны, преподавание в Гарварде обязательно имеет к этому какое-то отношение.

Может случиться так, что наиболее подготовленные ученики с хорошими связями, окончившие среднюю школу, в конечном итоге станут лучшими из тех, кто закончит колледж, независимо от того, в какой колледж они учатся. Другими словами, нам необходимо сфокусировать нашу оценку на том, сколько стоимости было добавлено организацией. Нам нужно знать, где студенты стояли в начале, чтобы оценить инструкции, которые они получили на своем пути от начала до конца. Нам необходимо предварительное и последующее тестирование и оценка, чтобы увидеть, какие школы, какие учреждения, какие округа действительно добавляют ценность и значительную ценность в качество мышления и обучения своих учеников.

Наконец, мы должны понять, что у нас уже есть инструменты для оценки того, что можно было бы назвать тонкоструктурными микро-навыками критического мышления.Мы уже знаем, как составлять подсказки, которые проверяют способность студентов определять правдоподобное утверждение о намерениях автора; четко различать цели; выводы, предположения и последствия; разумно обсудить достоинства разных версий проблемы или вопроса; решить наиболее разумное изложение авторской точки зрения; признать предвзятость, ограниченность и противоречия в точке зрения отрывка; отличать доказательства от выводов, основанных на этих доказательствах; давать показания в поддержку своей позиции в эссе; признать выводы, выходящие за рамки доказательств; отличать центральные понятия от периферийных; определить важнейшие последствия отрывка; оценивать выводы автора; сделать разумные выводы из заявленных позиций. . . и так далее.

Что касается интеллектуальных стандартов, мы вполне можем разработать подсказки, требующие от студентов распознавания ясности в отличие от нечеткости; отличать точные счета от неточных; решить, когда утверждение актуально или не имеет отношения к данному вопросу; выявлять непоследовательные и непоследовательные позиции; отличать глубокие, полные и важные отчеты от поверхностных, фрагментарных и тривиальных; оценивать ответы на предмет их справедливости; отличать убедительные свидетельства от не подкрепленных причинами и доказательствами; и отличить хорошие причины от плохих.

Что касается крупномасштабной оценки эссе, то теперь мы знаем достаточно о случайной выборке, чтобы иметь возможность требовать расширенного рассуждения и написания без необходимости платить за индивидуальную оценку миллионов эссе.

Остается применить то, что мы знаем, на практике: на уровне школы и округа, чтобы способствовать долгосрочному развитию учителей на основе мышления более высокого порядка, на уровне штата и страны, чтобы обеспечить долгосрочную оценку района, штата, и национальное исполнение. Проект займет поколения и, возможно, в каком-то смысле никогда не закончится.

В конце концов, когда мы достаточно далеко разовьем свое мышление, когда у нас будет достаточно интеллектуальной целостности, достаточно интеллектуальной смелости, достаточно интеллектуальной настойчивости, достаточно интеллектуальных навыков и способностей, достаточно справедливости, достаточно разумности?

Уж больно ясно одно. У нас уже более чем достаточно механического запоминания и скучного дидактического обучения; более чем достаточно пассивности и равнодушия, цинизма и пораженчества, самоуспокоенности и бездарности.Мяч на нашей стороне. Давайте вместе примем вызов и вместе с нашими учениками создадим новый, лучший мир.

{Это взято из книги: Как подготовить учеников к быстро меняющемуся миру Ричарда Пола.}

Вопросы и ответы по свободной практике

Тесты на когнитивные способности , например Поскольку тест Wonderlic, тесты Revelian и тесты Predictive Index стали важной частью процессов найма во многих компаниях. Они представляют собой форму психометрического теста, предназначенного для измерения интеллекта с помощью упражнений на логику, рассуждение и решение проблем.

В этой статье будет представлен широкий обзор того, что такое тесты на когнитивные способности, как они структурированы и как к ним подготовиться.

Краткая история тестов когнитивных способностей

Тесты на познавательные способности начали разрабатываться в конце 19 века как способ измерения «общих умственных способностей». Первоначально такие тесты были крайне неточными, в результате психологи разработали стандартизированных методов качественной оценки интеллекта и сравнения результатов тестов.

Например, психолог Уильям Стерн ввел термин «коэффициент интеллекта» в 1912 году как средство определения разницы между умственным возрастом ребенка и его хронологическим возрастом.

В 1904 году психолог Чарльз Спирман признал, что люди, продемонстрировавшие способность выполнять одну задачу, например, выявлять закономерности, также хорошо справляются с другими задачами, например, с решением арифметических задач. Спирмен предположил, что люди обладают «общими умственными способностями», подобными интеллекту.Таким образом, начала развиваться концепция теста для оценки когнитивных способностей.

После новаторской работы психологов, таких как Спирмен и Стерн, тесты когнитивных способностей стали общими инструментами набора во многих отраслях, от военных до продаж и всего, что между ними.

В США около 43% всех компаний сейчас используют психометрических теста, таких как когнитивные оценки, для определения пригодности кандидата на работу, а для компаний FTSE 100 этот показатель составляет 70% .

Поэтому весьма вероятно, что потенциальный работодатель попросит вас пройти тест на когнитивные способности.

Как работают тесты на когнитивные способности?

В классическом тесте когнитивных способностей используются вопросы следующих типов:

  • Численное мышление вопроса проверяют вашу способность понимать, анализировать и применять числовые и статистические данные. Вам нужно будет рассчитать проценты, заполнить недостающие числовые данные или определить следующее число в серии.
  • Устное мышление вопроса проверяют вашу способность понимать письменную информацию и использовать критический анализ. Классические вопросы потребуют от вас прочитать отрывок, а затем указать, являются ли утверждения по поводу отрывка «истинными», «ложными» или «не могу сказать».
  • Абстрактное мышление вопроса проверяют вашу способность работать с абстрактными идеями и концепциями. Вопросы часто включают визуальные диаграммы, которые вы должны использовать, чтобы определить недостающую информацию или завершить последовательность.
  • Пространственная осведомленность вопроса проверят вашу способность работать с узорами и формами. Часто задаваемые вопросы включают мысленное перестановку форм для создания новых или визуализацию узоров и изображений, когда они поворачиваются или переворачиваются.
  • Механическое мышление вопросов проверяют вашу способность использовать основные принципы механики, такие как работа с зубьями, рычагами, пружинами и шкивами.

Практический тест на познавательные способности

Большинство тестов можно выполнить с помощью компьютера.Обычно тест состоит из вопросов с несколькими вариантами ответов различной сложности; результаты представят точный профиль ваших интеллектуальных способностей.

Обычно тесты когнитивных способностей имеют ограничение по времени для завершения. Некоторые потребуют от вас ответа на все вопросы; другие попросят вас ответить на как можно больше вопросов в отведенное время.

Вы всегда должны знать, как будет рассчитан ваш конкретный тест во время подготовки.Имейте в виду, что время, необходимое вам для прохождения теста, может быть учтено в ваших результатах.

Почему работодатели используют тесты на когнитивные способности?

Психологи рекламируют тесты когнитивных способностей как отличные средства прогнозирования будущей производительности будущего сотрудника на работе. Тесты измеряют такие способности, как:

  • Понимание концепций
  • Абстрактное мышление
  • Решение проблем
  • Планирование и организация
  • Быстрое обучение на собственном опыте
  • Адаптация к неизвестным ситуациям
  • Применение новых знаний

Демонстрация высоких когнитивных способностей указывает на то, что кандидат хорош в адаптации к новой рабочей среде, принятии разумных решений и быстром обучении новым навыкам — необходимые навыки для достижения успеха на новой работе.

Кандидаты с более высокими баллами на тестах, как правило, более продуктивны и требуют меньше обучения, чем их коллеги с более низкими баллами. Это может означать значительные финансовые выгоды для работодателя.

По этим причинам тесты когнитивных способностей являются решающим, а иногда и решающим фактором при найме на работу многих работодателей.

Если вы планируете пройти когнитивный тест CCAT, посмотрите это видео от JobTestPrep, чтобы получить несколько полезных советов.



Пройти тест сейчас

Типы тестов когнитивных способностей

Работодатели получают свои тесты когнитивных способностей у различных поставщиков тестов в зависимости от их индивидуальных потребностей.Вот некоторые из поставщиков тестов и компаний, которые их используют:

Тест Wonderlic

Тест Wonderlic оценивает способности кандидатов на работу, используя сочетание логики и распознавания головоломок, распознавания фактов, задач со словами и вопросов вербального рассуждения.

Существует две версии теста:

  • The Wonderlic Personnel Test: 50 вопросов с несколькими вариантами ответов продолжительностью 12 минут.
  • The Wonderlic Personnel Test — QuickTest: 30 вопросов с несколькими вариантами ответов с ограничением по времени 8 минут.

Некоторые компании, которые, как известно, используют Wonderlic:

  • MENSA International
  • ThoughtWorks
  • Коммерческая группа побережья Мексиканского залива
  • Apple Шевроле

Тест на индекс прогнозирования

Когнитивная оценка Predictive Index проверяет такие навыки, как вербальное мышление, числовое мышление и распознавание образов. Он состоит из 50 вопросов, на которые нужно ответить за 12 минут.

Компании, использующие тесты Predictive Index:

  • ИКЕА
  • Шеврон
  • Microsoft
  • Dell

SHL Тесты

SHL — один из ведущих поставщиков тестов на профессиональную пригодность.Каждый тест SHL предназначен для оценки одной конкретной компетенции.

Тест обычно проводится в два этапа. Сначала кандидат проходит онлайн-тест. Затем, если кандидата приглашают на день оценки, ему будет предложено пройти еще одну более короткую версию теста, чтобы проверить свои ответы.

Типы тестов SHL включают:

  • Тест на вербальное мышление — 30 вопросов, на которые нужно ответить за 19 минут, или 18 вопросов за 11 минут, в день оценки.
  • Тест численного мышления — 18 вопросов, на которые нужно ответить за 25 минут, или 10 вопросов за 15 минут, в день оценки.
  • Индуктивный тест на рассуждение — 24 вопроса за 25 минут; предназначен для оценки ваших логических навыков.
  • Тест дедуктивного мышления — 20 вопросов, которые нужно выполнить за 18 минут. Требует от вас использования логики, чтобы делать выводы, выявлять ошибки в информации и оценивать аргументы.

Компании, использующие тесты SHL, включают:

  • Филип Моррис
  • Компания Gannett
  • Форд Мотор
  • Microsoft

Ревельские испытания

Ревельских тестов сдают более 200 000 человек по всему миру, и это самый популярный психометрический тест, который сдают выпускники в Австралии.

Ревелиан определил различные характеристики, типичные для кандидатов с когнитивными способностями, необходимыми для достижения успеха в работе, и разработал тесты способностей в следующих областях:

Компании, использующие тесты Revelian, включают:

Как подготовиться к тесту на когнитивные способности

Многие кандидаты считают, что подготовка к тестам на когнитивные способности бесполезна. Некоторые думают, что их интеллект говорит сам за себя; другие думают, что, поскольку вопросы случайны и их невозможно предсказать, подготовиться к ним невозможно.

На самом деле, вы можете (и должны) подготовиться к тестам на когнитивные способности, ознакомившись с:

  • Структура теста.
  • Временной интервал , в котором необходимо пройти тест.
  • типов вопросов, которые появятся .

Чем больше вы знакомы с общим форматом теста, тем больше времени вы сможете потратить на ответы на вопросы в отведенное время.

Есть также бесчисленное множество примеров когнитивных способностей , примерных вопросов и тестов, к которым вы можете получить доступ в Интернете, чтобы подготовиться к тесту.Хотя вопросы не будут такими же, как в реальном тесте, лучше всего вы сможете подготовиться, практикуя аналогичные стили вопросов и рассчитывая время самостоятельно.

Если вы ищете бесплатных образцов когнитивных тестов , вот вам бесплатный образец численного практического теста и бесплатный образец вербального теста на рассуждение, чтобы вы могли начать.

Вы также можете найти практические тесты на таких сайтах, как Job Test Prep.

Поскольку большинство практических тестов не рассчитаны по времени, установите таймер на одну минуту на каждый вопрос , чтобы ваша практика как можно точнее отражала истинные условия тестирования.

Больше практики

Тестовый день: советы по проведению самого теста

  • Высыпайтесь за ночь за ночь перед тестом и съешьте хороший завтрак , чтобы быть в отличной форме.
  • Пейте много воды и не забудьте взять воду на тест (если он проводится в контролируемых условиях).
  • Определите максимальное количество времени, которое вы можете потратить на каждый вопрос (например, одну минуту на вопрос).
  • Возьмите секундомер , чтобы рассчитать время и не тратить слишком много времени на каждый вопрос.
  • Если вы проходите тест онлайн, приготовьте блокнот или листок бумаги и ручку для задач, которые вы не можете решить в своей голове.
  • Прочтите каждый вопрос дважды и постарайтесь не отвечать слишком быстро . Это особенно важно в тестах на вербальное мышление — обратите внимание на отрицательные, двойные отрицательные или другие изменяющие слова, которые могут сбить вас с толку, если вы скользите по ним.
  • Не применяйте посторонние знания к вопросам — вся информация, необходимая для логического ответа, будет в вопросе.

Последние мысли

Несколько последних советов, о которых следует помнить:

  • Спросите потенциальных работодателей, какие тесты когнитивных способностей им требуются как часть процесса подачи заявок, чтобы вы могли начать подготовку.
  • Практика, практика, практика . Воспроизведите истинные условия тестирования, выбрав время проведения тестов.
  • Проверьте, разрешено ли вам делать заметки во время теста . Если нет, потренируйтесь отвечать на типовые вопросы, используя только свою память и умственные способности.
  • Не тратьте много времени на каждый вопрос , так как это может повлиять на вашу общую оценку.

Наконец, попробуйте эти полезные практические тесты, которые помогут вам почувствовать настоящую вещь.

Практический тест на познавательные способности

Психологические установки и видение решений проблем

Ментальный набор — это тенденция видеть только те решения, которые работали в прошлом. Этот тип навязчивого мышления может затруднить поиск решений и затруднить процесс решения проблем. Например, представьте, что вы пытаетесь решить математическую задачу на уроке алгебры. Проблема кажется похожей на те, над которыми вы работали ранее, поэтому вы подходите к ее решению таким же образом. Из-за вашего мышления вы не можете найти более простого решения, которое могло бы быть возможным.

Обзор

Когда мы решаем проблемы, мы часто склонны прибегать к решениям, которые работали в прошлом.Во многих случаях это полезный подход, позволяющий быстро находить ответы. Однако в некоторых случаях эта стратегия может затруднить поиск новых способов решения проблем.

Влияние прошлого опыта

Хотя во многих случаях мы можем использовать наш прошлый опыт для решения проблем, с которыми мы сталкиваемся, это может затруднить поиск новых или творческих способов решения текущих проблем. Например, представим, что ваш пылесос перестал работать. .Когда в прошлом он переставал работать, причиной был обрыв ремня. Поскольку прошлый опыт научил вас, что ремень — обычная проблема, вы снова меняете ремень, но на этот раз вакуум продолжает работать со сбоями.

Вы просите друга прийти посмотреть на пылесос, и он обнаруживает, что одно из шланговых приспособлений не было подключено, что привело к потере всасывания вакуума. Из-за своего психологического настроя вы не заметили достаточно очевидного решения проблемы.

Однако, как вы могли догадаться, ментальные установки также могут создавать проблемы, как большие, так и мелкие.

Функциональная неподвижность

Функциональная фиксированность — это особый тип ментальной установки, предполагающий только способность видеть решения, предполагающие использование объектов их обычным или ожидаемым образом. Иногда могут быть полезны мысленные установки. Используя стратегии, которые работали раньше, мы часто можем быстро находить решения. Это может сэкономить время, и во многих случаях такой подход действительно дает правильное решение.

В повседневной жизни психологическая установка может помешать вам решить относительно небольшую проблему (например, выяснить, что не так с вашим пылесосом).В более широком масштабе ментальные установки могут помешать ученым найти ответы на реальные проблемы или затруднить врачу определение причины болезни.

Например, врач может увидеть нового пациента с симптомами, аналогичными определенным случаям, которые он видел в прошлом, поэтому он может поставить диагноз этому новому пациенту с тем же заболеванием. Из-за такой психологической установки врач может упустить из виду симптомы, которые на самом деле вообще указывают на другое заболевание. Очевидно, что такие ментальные установки могут иметь огромное влияние на здоровье пациента и возможные результаты.

интеллектуальных игр: тренировка для ума | Здоровье и благополучие

Шарон, 46-летняя мать-одиночка троих подростков, пришла ко мне по поводу своей возрастающей забывчивости. Работа полный рабочий день и ведение домашнего хозяйства становились для нее непосильными, она теряла ланчбоксы, пропускала встречи и не могла сосредоточить свое внимание. Она волновалась, потому что ее бабушка заболела болезнью Альцгеймера в возрасте 79 лет, и Шэрон чувствовала, что, возможно, она тоже заболела — только намного моложе.Я сказал, что маловероятно, что Шэрон страдает слабоумием на ранней стадии, но согласился провести ее обследование.

Всякий раз, когда я консультируюсь с людьми по поводу их перерывов в работе среднего возраста, я сначала проверяю их физическое состояние или побочные эффекты лекарств, которые могут повлиять на здоровье их мозга. При отсутствии лечения повышенный уровень холестерина, гипертония и другие возрастные заболевания могут ухудшить память, повысить риск деменции и сократить продолжительность жизни. Я также просматриваю их повседневный образ жизни, чтобы узнать, есть ли какие-то области, которые они могут улучшить, чтобы улучшить здоровье своего мозга.

Я убедил Шэрон записаться в двухнедельный исследовательский проект в Центре долголетия Калифорнийского университета в Лос-Анджелесе, чтобы определить влияние программы здорового образа жизни на мозг. Перед начала программы, ее оценка памяти базисной была около среднего для своего возраста, и МРТ сканирование во время задач памяти показало обширную нейронную активность — ее мозг работало трудно запомнить вещи.

Затем Шэрон начала программу ежедневных физических упражнений, тренировки памяти, здорового питания и упражнений на расслабление.Через две недели ее тесты памяти продемонстрировали значительные улучшения, а последующая МРТ показала минимальную нервную активность во время запоминания слов — ее мозг стал более эффективным. Шэрон была поражена тем, что всего за две недели у нее улучшилась память и стало легче узнавать и извлекать новую информацию. Ее здоровая диета и упражнения помогли Шэрон сбросить несколько лишних килограммов, и она чувствовала себя более уверенно — как дома, так и на работе. Эти поразительные улучшения побудили ее на протяжении многих лет поддерживать новые здоровые привычки мозга.

Опыт Шэрон был аналогичен опыту многих других, кто участвовал в наших программах здорового образа жизни и протоколах исследований. Наша команда показала, что двухнедельная программа, сочетающая умственные и физические упражнения, снижение стресса и здоровое питание, связана со значительным влиянием на когнитивные функции и метаболизм мозга, которые мы измерили с помощью сканирования с помощью ПЭТ (позитронно-эмиссионной томографии). Мы наблюдали снижение активности лобной доли мозга, что может отражать более высокую когнитивную эффективность области мозга, участвующей в рабочей памяти, форме кратковременной памяти, связанной с немедленной когнитивной обработкой и решением проблем.

К сожалению, несмотря на убедительные научные данные о том, что повседневное поведение может улучшить память и снизить риск деменции, большинству людей сложно изменить старые привычки. Однако задача становится намного проще, когда люди понимают связь между повседневным поведением и здоровьем мозга, ставят разумные цели и получают обратную связь, которая их мотивирует.

В сотрудничестве с организацией Gallup Poll наша исследовательская группа из Калифорнийского университета в Лос-Анджелесе оценила ответы более чем 18 000 человек в возрасте от 18 до 99 лет о поведении, которое поддерживает память. Чем большее количество привычек здорового образа жизни (например, физические упражнения, здоровое питание, отказ от курения) практиковали люди, тем лучше были их показатели памяти. Респонденты, придерживавшиеся только одного здорового образа жизни, на 21% реже сообщали о проблемах с памятью, в то время как те, кто придерживался трех видов здорового образа жизни, на 75% реже замечали забывчивость. Эти результаты согласуются с другими исследованиями, показывающими синергетические преимущества сочетания здорового образа жизни для снижения риска диабета и сердечных заболеваний.

Оцените свой базовый уровень памяти

В моей книге «2 недели до молодого мозга» Джиджи Ворган и я описываем научные доказательства того, что привычки образа жизни имеют значение, когда речь идет о здоровье мозга, а также 14 дней стратегий и упражнений для читателей. начали вести здоровый для мозга образ жизни, который они могут продолжать всю оставшуюся жизнь. Вы можете попробовать это сами, оценив свою базовую память и снова проверив ее после изучения некоторых техник.

Ниже приведен список из восьми не связанных между собой слов.Установите секундомер на одну минуту, чтобы изучать слова. Затем потратьте 10 минут на что-нибудь другое. Затем запишите столько слов, сколько сможете запомнить, чтобы получить базовую оценку памяти:

  • Лошадь.

  • Клоун.

  • Дерево.

  • Доктор.

  • Трубка.

  • Гитара.

  • Оранжевый.

  • Стул.

Методы памяти, повышающие мощность мозга

Научиться создавать визуальные образы, которые представляют информацию, которую вы хотите вспомнить позже, поможет резко активизировать ваши способности памяти.Я рекомендую один метод запоминания, метод рассказа, полезен для запоминания списков предметов и поручений, когда вы в пути. Создавая историю, которая связывает ваши несвязанные слова, ваши визуализации и ассоциации автоматически улучшат вашу память. Посмотрите еще раз на восемь слов и на этот раз потратьте минуту на создание истории, которая связывает их все вместе. Возможно, вы визуализируете клоуна с оранжевыми волосами верхом на лошади или доктора, курящего трубку и играющего на гитаре. Используйте свои первые ассоциации и связывайте изображения в любом порядке.Теперь напишите слова еще раз и посмотрите, насколько улучшилась ваша память.

Этот метод использует естественную способность человеческого мозга замечать и запоминать визуальные подсказки — навык, который развивался за миллионы лет. Сочетание визуализации с ассоциациями сделает ваши воспоминания более значимыми и, следовательно, более запоминающимися. Эта стратегия полезна при наиболее распространенных возрастных жалобах на память, связанных с забыванием имен и лиц.

Умственная стимуляция с помощью игр, головоломок, чтения и разговоров связана с улучшением памяти, поэтому я призываю людей сохранять активный ум.

Игры для мозга

На протяжении многих лет мы знали, что физические упражнения укрепляют наше тело, и теперь научные данные свидетельствуют о том, что умственные упражнения сохраняют молодость нашего мозга.

Моя двухнедельная программа «Мозг младшего возраста» включает игры и упражнения, которые активируют ваши нейронные цепи и укрепляют ваши умственные способности. По мере того, как вы постепенно совершенствуете свои навыки, вы обнаружите, что эти игры становятся проще, поэтому вы можете постепенно повышать уровень сложности.

Ниже приведены некоторые примеры игр для мозга, которые улучшают ряд когнитивных функций, помогая сохранять гибкость и остроту ума.У правшей левая часть мозга обычно контролирует речь и мыслительные способности. Игры в слова помогают строить это левое полушарие, в то время как лабиринты и головоломки укрепляют правое полушарие, которое контролирует зрительные навыки и ориентацию. Ответы ниже.

Упражнения для начинающих

1. Упражнение для правого мозга: счет квадратов

Подсчитайте количество квадратов на рисунке слева. Подсказка: Обязательно посчитайте квадраты внутри квадратов.

2. Левое полушарие : изменение слов

Начните со слова СТЕНА и меняйте букву за раз, пока не получите слово ФИРМА. Каждое изменение должно быть правильным словом.

СТЕНА

_ _ _ _

_ _ _ _

_ _ _ _

ФИРМА

3. Правое полушарие : зубочистки для нумерации

Разложите три зубочистки (без зубочисток) ломая или сгибая их).

4.Левое полушарие : битва букв № 1

Составьте как можно больше слов из следующих букв. Используйте каждую букву только один раз в каждом слове.

IRNAB

5. Правое полушарие мозга : головоломка «мозговой разрыв»

Какой фрагмент подходит для головоломки?

Упражнения среднего уровня

6 . Правое полушарие : непрерывная линия

Улучшает зрительно-пространственные навыки и способность лобной доли распределять ваше внимание между умственными задачами.

На рисунке слева нарисуйте непрерывную линию, соединяющую число 1 с буквой A, затем A с 2, затем 2 с B, затем B с 3 и так далее, пока вы не сможете продолжить числовой или алфавитный последовательность.

7 . Левое полушарие : поиск цветов

Переставьте все буквы, чтобы найти четыре цвета, смешанные ниже. Подсказка: только один цвет является основным.

RAIGET

ENOLYL

OVGOEN

LEWRE

8.Левое полушарие : пословица

В следующей пословице удалены все гласные, а оставшиеся буквы сгруппированы в группы по три или четыре буквы в каждой. Замените гласные и откройте пословицу.

TWH

DSRB

TTRT

HNN

Расширенные упражнения

Теперь вы достаточно разогреты, чтобы начать использовать весь свой мозг (как правое, так и левое полушария), чтобы попытаться решить эти головоломки.

9 . Буквенная схватка № 2

Попытайтесь придумать как можно больше слов (две или более букв) из следующего: ОГЕУНРИ

Дополнительный кредит: Семибуквенное слово из этой буквенной схватки и пятибуквенное Слово из буквенной схватки №1 напомнит вам название моей книги.

10 . Финики Фрэнк

У Фрэнка очень эксцентричные вкусы. Он фанат футбола, но ненавидит регби; любит пиво, но ненавидит эль; ездит на Феррари, но его не поймают мертвым на Ламборджини.Судя по привередливым вкусам Фрэнка, он предпочел бы кататься на лыжах или велосипеде?

Ответы

1. 21 квадрат.
2. СТЕНА, ВОЛЯ, ЗАЛИВКА, ПЛЕНКА, ФИРМА.
3. IX (римская цифра).
4. Я, Ин, Ран, Ребро, Дождь, Наб, А, Ан, Воздух, Бан, Бар, Бин, Сарай, Бран, Мозг.
5. Б.
6. Форма звезды.
7. Зеленый, оранжевый, фиолетовый, желтый.
8. Две головы лучше, чем одна.
9. Он, Или, Один, Руда, Онер; Go, Gun, Guy, Gone, Gore, Gray, Goner; Rue, Run, Rug, Rung, Rouge; Ты, молодой, младший. Слово из семи букв — Младший, а слово из пяти букв — Мозг.
10. Катание на лыжах, так как ему нравятся только слова, содержащие двойные буквы.

• Д-р Гэри Смолл — профессор психиатрии и старения и директор Центра долголетия в Институте неврологии и поведения человека Семела Калифорнийского университета, Лос-Анджелес

Пазлы перепечатаны с разрешения, все права защищены , 2 недели младшему мозгу © Humanix Books, 2014

• 17 октября 2018 г. в эту статью были внесены поправки, в которых слово «Nan» было удалено из ответа на вопрос 4.

Интеллектуальные навыки критических мыслителей

Людей:

В сообщении ниже исследуются семь интеллектуальных привычек критически мыслящих людей. Он взят из рисунка 4.1 «Интеллектуальные привычки критических мыслителей» из главы 4 «Устранение барьеров для критического мышления» книги Эдмунда Дж. Хансена «Обучение на основе идей: процесс разработки курса для содействия концептуальному пониманию». Издано ООО «Стилус Паблишинг». 22883 Quicksilver Drive, Стерлинг, Вирджиния 20166-2102 www.styluspub.com. Авторские права © 2011 ООО «Стилус Паблишинг». Все права защищены. Печатается с разрешения.

С уважением,

Рик Рейс

[email protected]

UP NEXT: Коучинг и обучение

Преподавание и обучение завтрашнего дня

————————————— 863 слова ——— —————————

Интеллектуальные навыки критических мыслителей

Беспристрастность влечет за собой осознание необходимости одинаково относиться ко всем точкам зрения, независимо от собственных чувств или эгоистических интересов.Он основан на осознании того факта, что мы по своей природе склонны предвзято относиться к взглядам других, помещая их в категории \ «благоприятные \» (согласны с нами) и \ «неблагоприятные \» (не согласны с нами). Мы склонны уделять меньше внимания противоположным взглядам, чем нашим собственным. Объективность требует от нас развития:

1. Интеллектуальное смирение

Осведомленность о своих предубеждениях, предрассудках, ограничениях своей точки зрения и степени своего невежества. (Например, многие американские и другие западные студенты считают свой образ жизни — соревнование, индивидуализм, материализм, демократические формы правления, устройство нуклеарной семьи, трудовую этику — превосходящими незападные ценности и условия жизни.Их предубеждения оказывают глубокое влияние на их понимание важных концепций в социальных науках, искусстве и гуманитарных науках.)

2. Интеллектуальное мужество

Сознание необходимости встретиться лицом к лицу с идеями, убеждениями или точками зрения, в отношении которых у человека есть сильные отрицательные эмоции и которые не были всерьез выслушаны, и справедливо рассмотреть их; признание того, что идеи, которые общество считает опасными или абсурдными, иногда рационально оправданы — полностью или частично (например,, В любой культуре есть свои табу, которые также влияют на научный дискурс. Недавние примеры включают исследование стволовых клеток, однополые браки, мусульманский радикализм или любой другой радикализм по этому вопросу, глобальное потепление, атеизм, позитивные действия, помощи самоубийству, и порнографию. Требуется смелость, чтобы открыто исследовать любые потенциальные рациональные корни любого из этих противоречивых форм поведения и убеждений.)

3. Интеллектуальная эмпатия

Осознание необходимости творчески поставить себя на место других, чтобы по-настоящему понять их.(Старые парадигмы в социальных науках часто относились к \ «предметам \» своих исследований как к переменным, на которые следовало смотреть без эмоционального участия, чтобы гарантировать \ «объективность \». В настоящее время многие социологи используют другой подход к пониманию Социальная среда. Чтобы полностью понять поведение и намерения других, молодые ученые должны научиться воспринимать точку зрения своих субъектов исследования, что требует определенной степени личной идентификации, которую ранее осуждали как загрязнение исследовательского процесса. Подобные способности всегда считались предпосылкой для создания и оценки хорошей литературы и других видов искусства.)

4. Интеллектуальная честность

Признание необходимости быть верным собственному мышлению и придерживаться тех же стандартов, которых ожидают от других. Это также означает честно признавать несоответствия и несоответствия в собственных мыслях и действиях. (например, сглаживание углов, плагиат и обман стали широко распространенными не только в колледже, но и в аспирантуре и за ее пределами.Ожидания общества относительно ускорения результатов во всех сферах жизни, в том числе в академических кругах, могут оказать огромное давление на студентов, чтобы они впечатляли своей производительностью за счет академической строгости и актуальности. Признание недостатков в своем мышлении требует такого же мужества, как и справедливое рассмотрение точек зрения, с которыми человек категорически не согласен; см. пункт 2.)

5. Интеллектуальная настойчивость

Склонность преодолевать интеллектуальные сложности, несмотря на разочарование, присущее задаче. (Многие учащиеся в нашей нынешней школьной системе учатся избегать тех вещей, которые кажутся слишком сложными: \ «инженерия слишком утомительна \» \ «математика слишком сложна и \» степень доктора бухгалтерского учета не окупается \ «. ибо плоды своего труда ученику так же тяжело, как ребенку ждать десерта. Это относится также и к повседневной борьбе с интеллектуальными задачами. Многие ученики просят простых ответов и с подозрением относятся к их дисциплине ответы на сложные вопросы, или когда эти ответы остаются неоднозначными.)

6. Доверие к разуму

Вера в то, что собственные высшие интересы и интересы человечества в целом будут наилучшим образом удовлетворены, если будет максимально свободно играть разум, поощряя людей делать свои собственные выводы, развивая свои собственные рациональные способности; вера в то, что при должном ободрении и совершенствовании люди могут научиться думать самостоятельно. (Уверенность в разуме — это также уверенность в других. Это педагогический принцип, которым руководствуются хорошие учителя. Учащимся не следует убеждать принять точку зрения своих учителей или учить подходить к задачам только одним определенным образом.Сложное понимание необходимо развивать, а не принуждать. Обретение свободы и поощрения решать проблемы по-своему помогает создать интеллектуальную зрелость. Это включает в себя свободу делать собственные ошибки и учиться на них.)

7. Интеллектуальная автономия

Внутренняя мотивация, основанная на идеале мышления для себя; рациональное авторство своих убеждений, ценностей и образа мышления; не зависеть от других в отношении направления и контроля своего мышления.(Традиционная парадигма обучения, говорящая студентам о том, что нужно изучать с помощью лекций и учебников, превратила студентов в пассивных получателей знаний. Учителя были экспертами, которым студенты доверяли всегда иметь правильные ответы. Не нужно думать самостоятельно. Новая парадигма обучения ставит студентов в контроле и заставляет их отвечать за собственное обучение.Теория обучения обнаружила различные стили обучения, а теория мотивации показывает, что глубокое понимание связано с автономией учащегося. Чем более уверенными учащиеся будут находить свое собственное направление, тем больше у них шансов выработать комплексное понимание предметов своего обучения.)

Номер ссылки

Пол Р. и Элдер Л. (2001). Критическое мышление: инструменты, позволяющие взять на себя ответственность за свое обучение и свою жизнь. Река Аппер Сэдл, Нью-Джерси: Prentice Hall.

передовых тестов (+ Ultimate Guide)

Если вы хотите пройти бесплатный практический тест на когнитивные способности перед чтением этой статьи, нажмите здесь.

Если вы хотите приобрести онлайн-пакет для подготовки к тесту на когнитивные способности, посетите наш партнерский веб-сайт JobTestPrep.

Большинство из нас было приглашено хотя бы один раз для прохождения какого-либо теста перед приемом на работу в рамках процесса найма и отбора.

Одной из наиболее часто используемых работодателями оценок являются тесты когнитивных способностей (обычно известные как тесты коэффициента интеллекта (IQ) или оценки общих способностей (g)), поскольку они, как известно, влияют на производительность сотрудников и прогнозируют ее.

Что такое когнитивные способности?

Gottfredson (1997) дал широкое и всеобъемлющее определение когнитивных способностей как:

«очень общие умственные способности, которые, среди прочего, включают способность рассуждать, планировать, решать проблемы, мыслить абстрактно, понимать сложные идеи. , быстро учиться и учиться на собственном опыте. Это не просто книжное обучение, узкие академические навыки или умные тесты. Скорее, он отражает более широкую и глубокую способность к пониманию нашего окружения — «улавливать», «понимать» вещи или «выяснять», что делать.” (стр. 13)

Компании вкладывают огромные средства (т.е. деньги, время и ресурсы) в инструменты, которые могут улучшить их процесс найма.

Они делают это, потому что многие исследования показывают, как оценка талантов, актуальная для работы, приводит к значительному повышению производительности, повышению производительности, экономии средств и снижению текучести кадров.

Стоит отметить, что компании обычно используют оценки, которые: Действителен, (может предсказать эффективность работы), Надежен, (имеют результаты, согласованные с течением времени), Недискриминационный (Показатели групп меньшинств аналогичны другим группам) , и которые отформатированы таким образом, чтобы вызвать благоприятную реакцию кандидата .

СОВЕТ ПРОФЕССИОНАЛА: Посетите JobTestPrep , чтобы получить лучшую практику онлайн-психометрических тестов. Тестовые наборы начинаются с 39 фунтов стерлингов с гарантией возврата денег .

Когнитивные способности и производительность труда, выполнение задач, креативность и успех в карьере

Более чем за столетие исследований было установлено, что когнитивные способности являются сильным предсказателем эффективности работы , и хотя это верно для различных типов и уровней задач и заданий, вас с большей вероятностью попросят пройти тест когнитивных способностей, если роль, которую вы назначаете, требует выполнения сложных задач, поскольку валидность тестов на когнитивные способности увеличивается на с увеличением сложности задания / задачи.

Исследователь Кэмпбелл (1990b) объяснил этот факт, сказав: «Общие умственные способности являются фундаментально значимым предиктором индивидуальных различий в производительности труда для любой работы, которая включает обработки информации задач».

Он добавил, что «точный размер отношений будет зависеть от […] степени, в которой работа требует обработки информации и вербальных когнитивных навыков».

Соответственно, на таких должностях, как инженеры, врачи, адвокаты и пилоты, взаимосвязь между оценками когнитивных способностей и общей производительностью труда оказывается наиболее сильной.

Помимо общей производительности труда, работодатели обычно заинтересованы в том, чтобы узнать, как вы будете выполнять конкретные технические задачи, обычно называемые . Производительность задач , на которую также влияет то, насколько сильны ваши интеллектуальные способности.

Кроме того, если вы набрали высокие баллы по тестам когнитивных способностей, вы, вероятно, станете более эффективным лидером (Judge, Colbert, & Ilies, 2004), более креативным и новаторским (Kuncel et al. , 2004), с меньшей вероятностью уйдете. компании (Maltarich, Nyberg, and Reilly, 2010) и продемонстрирует более высокий уровень внешнего карьерного успеха (получать больше заработной платы со временем и получать более частые продвижения по службе), как пришли к заключению Нг, Эби, Соренсена и Фельдмана в исследование, которое они провели в 2005 году.

Измерение когнитивных способностей

Как вы видели, учитывая их надежный прогноз производительности (и относительно низкую стоимость для организаций), неудивительно видеть, что компании просят вас пройти оценку когнитивных способностей в рамках их программ набора и отбора.

Эти тесты интеллекта или когнитивных способностей разработаны для формального изучения — в заранее определенных условиях — того, насколько хорошо вы можете адаптироваться, приспосабливаться и извлекать уроки из конкретной ситуации.

Эти тесты являются частью так называемых «психометрических оценок», которые используются для измерения психических аспектов; где слово «метрика» означает меру, а слово «псих» означает психическое.

Как мы обсуждали в других статьях, существует широкий спектр психометрических оценок, которые можно использовать для измерения почти каждого угла психического или эмоционального состава человека, включая личность, отношение и интеллект.

Однако, несмотря на их вариации, большинство этих тестов можно разделить на две основные категории, которые часто используются вместе, чтобы дать работодателям целостное представление о вашей пригодности как потенциального сотрудника; 1- Типовые тесты производительности (e.грамм. анкеты личности, раскрывающие детали вашей личности и предпочтений) и 2- Тесты максимальной производительности и потенциала (тесты способностей и способностей).

Не вдаваясь в подробности технического характера, слово «способность» часто неверно истолковывают как отражение способностей или достижений, а в контексте психометрической оценки способности можно рассматривать как еще один способ обозначения конкретных способностей.

Однако существует тонкая техническая разница между «достижением / достижением», «способностью» и «способностью», которую можно объяснить следующим образом:

  • Достижение: То, чего вы достигли в прошлом.
  • Способность: Что вы способны продемонстрировать в настоящем.
  • Aptitude: Насколько быстро или легко вы сможете учиться в будущем.

В этой статье мы рассмотрим оценку когнитивных способностей (интеллекта) (как общих способностей, так и способностей) и предоставим вам несколько примеров типов вопросов, с которыми вы можете столкнуться, если будете проходить какой-либо из этих тестов.

Общий и множественный интеллект

Теория общих способностей / интеллекта (или g ) была представлена ​​в 1904 году английским психологом Чарльзом Спирменом, который пришел к выводу, что общая способность (g) является единственной общей мерой, позволяющей прогнозировать успешную успеваемость. широкий спектр задач, и хотя люди могут и обычно действительно проявляют себя в определенных / конкретных областях, он утверждал, что действительно существуют широкие умственные способности и что люди, которые хорошо работают в одной области (например,грамм. вербальные способности), вероятно, успешно справились бы с другими тестами.

Самая популярная оценка когнитивных способностей, основанная на теории общих способностей и используемая в течение многих лет, — это тест на коэффициент интеллекта (IQ).

Теория общих способностей часто оспаривается теми, кто утверждает, что концепция ограничения умственных способностей одним общим интеллектом слишком упрощена.

Кроме того, растет количество исследований, предполагающих, что умственные способности не только определяются ДНК человека, но также зависят от социальных факторов, таких как образование и социально-экономический уровень.

Это привело к появлению концепции множественного интеллекта (МИ), которая продвигает идею о том, что люди обладают множественным интеллектом, и, приняв определение когнитивных способностей, включающее различные типы интеллекта, можно будет идентифицировать, ценить и развивать ее / его сильные стороны.

Эта концепция множественного интеллекта (МИ) была впервые введена профессором Ховардом Гарднером из Гарварда, который определил интеллект как «потенциальную способность обрабатывать определенный вид информации» .

Он пришел к выводу, что разные типы интеллекта по большей части независимы друг от друга, и относительная важность одного типа над другим зависит от требований ситуации (или работы). Он резюмировал 7 типов интеллекта следующим образом:

  • Вербальный (лингвистический): лексические навыки, формальная речь, вербальные аргументы и творческое письмо.
  • Тело (кинестетика / движение): язык тела, физические жесты, творческий танец, физические упражнения, драма.
  • Музыкальный (ритмический): музыкальное исполнение, пение, музыкальная композиция и ритмические паттерны.
  • Логика (математика): способности к вычислению, решению задач и расшифровке кодов, абстрактных символов и формул.
  • Визуальный (пространственный): Разработка и распознавание узоров и рисунков, живопись, рисунок, активный образ, воображение, скульптура и цветовые схемы.
  • Межличностное общение (отношения с другими): Индивидуальное общение, практики сочувствия, групповые проекты, навыки сотрудничества, получение и предоставление обратной связи.
  • Внутриличностный (самопонимание и понимание): стратегии мышления, эмоциональная обработка, познание себя, рассуждение более высокого порядка, сосредоточение и концентрация.

Кроме того, Creativity и Memory иногда добавляются как отдельные интеллекты, составляя восьмую и девятую категории интеллекта согласно модели Гарднера.

Для проверки множественного интеллекта, определенного Гарднером, обычно проводятся специальные тесты способностей, и мы рассмотрим некоторые из тех, которые часто используются на рабочем месте, такие как вербальный интеллект, числовой интеллект, технические способности и логические рассуждения.

Тесты интеллекта (General Intelligence Theory, g)

  • Коэффициент интеллекта (IQ)

Коэффициент интеллекта (IQ) — это возрастной тест уровня интеллекта, который рассчитывается путем деления умственного возраста на их возраст и умножьте его на 100.

Слово «частное» означает результат деления одного количества на другое. Как мы уже упоминали, тесты IQ основаны на теории, что у вас есть один-единственный интеллект (или способность), который относится к общей способности (g).Эта общая способность определяет, насколько эффективно мы справляемся с проблемами и проблемами по мере их возникновения и как мы мысленно используем наш предыдущий опыт.

Эти умственные способности различаются от одного человека к другому, что является целью оценки интеллекта (тесты IQ).

IQ-тесты состоят из серии вопросов и задач, каждый из которых был стандартизирован для очень большой выборки с различными демографическими данными, что делает его надежным прогнозом для умственных способностей всех возрастов, этнических групп, полов и культур, а также В отличие от определенных навыков, тесты на IQ — это стандартные экзамены, предназначенные для измерения интеллекта в отдельности от его академических или профессиональных достижений.

Хотя можно найти много разных типов тестов IQ; стандартный тест будет состоять из трех частей, каждая из которых проверяет разные когнитивные способности; вербальное мышление, числовые способности и схематическое (или пространственное) мышление .

Чтобы тест IQ мог уверенно измерить общие общие способности или интеллект человека, вопросы обычно являются междисциплинарными и содержат смесь вербальных, математических и пространственных задач, а также дополнительные вопросы, которые оценивают, насколько логичен человек. мыслительные процессы связаны с тем, есть ли у них уровень латерального мышления или нет.

Ваши показатели IQ обычно стабильны с течением времени, и широко распространено мнение, что это связано с чьей-то ДНК и, следовательно, с трудностью значительно улучшить свой IQ со временем, хотя можно было бы — вопреки тому, что говорят многие люди. — через постоянную практику улучшить свои тестовые характеристики.

Практические тесты на IQ действительно работают, и если во время оценки вас попросят пройти оценку IQ и вы уже делали подобное раньше, вы, скорее всего, получите более высокие баллы в некоторых областях.

Конечно, практикуясь, вы не сразу наберете много очков, чем ваш средний результат, однако результат зависит от различных факторов, помимо вашего уровня интеллекта, таких как время, которое вы потратили на выполнение заданий, и от того, насколько вы нервничаете в это время. завершение теста.

Именно эти факторы вы можете улучшить, заранее потренировавшись в тестах IQ. Вы будете знать, чего ожидать, и благодаря этому будете меньше нервничать.

Кроме того, вы сможете выполнять задания немного быстрее, потому что вы узнаете типы вопросов.

Ниже приведены некоторые примеры вопросов, которые обычно задают в тесте IQ:

Дополнительные вопросы теста IQ см. В видео ниже:

Тесты интеллекта (теория множественного интеллекта, g)

Как мы до сих пор пришли к выводу, хотя могут быть разные определения интеллекта, тем не менее есть по крайней мере одно особенно подходящее основание; способность учиться и понимать .

Как мы видели в предыдущем разделе, результаты стандартизированных тестов интеллекта (баллы IQ) могут использоваться для определения уровня интеллекта, однако все больше становится общепризнанным, что оценки IQ не всегда рассказывают полную историю и могут лишь предоставлять снимок ваших когнитивных способностей.

Соответственно, отдельные тесты на вербальные, математические, логические и недавние творческие / новаторские и межличностные отношения (также известные как эмоциональный интеллект) часто используются организациями, потому что очевидно, что большее количество различных типов областей, которые проверяются и исследуются , тем точнее можно оценить уровень интеллекта человека.

На самом деле существует более 50 различных человеческих способностей, однако все они попадают в следующие четыре основные категории:

  1. Когнитивные способности: вербальные, числовые, абстрактные, перцепционные, пространственные, механические.
  2. Психомоторные способности: координация глаз и рук.
  3. Сенсорные способности: слух, осязание, осязание, обоняние, зрение.
  4. Физические способности: выносливости и силы.

Вы должны всегда помнить, что для большинства тестов на пригодность, которые вас попросят пройти, обычно есть заранее установленный срок, который необходимо строго соблюдать, чтобы тест был действительным, и обычно есть средний балл. который был стандартизирован по сравнению с группой людей со схожими демографическими данными, которые прошли такой же тест.

При выполнении в этих заранее заданных условиях ваши окончательные результаты обычно подпадают под один из следующих пяти уровней, выраженных в процентах относительно установленных средних баллов, соответствующих кривой нормального распределения:

  1. Лучшие 10% населения : Чрезвычайно высокий уровень способностей.
  2. Верхняя треть (исключая 10% лучших) — Высокий уровень способностей.
  3. Оценка, полученная одной третью населения — Средняя степень
    способностей.
  4. Самая низкая треть — Степень пригодности ниже среднего.
  5. Самая низкая 10% — Минимальная демонстрация этой способности.

Давайте теперь рассмотрим различные тесты способностей, которые могут дать некоторое представление об индивидуальном уровне интеллекта или могут быть использованы вместе, чтобы дать вашему будущему работодателю представление о ваших общих способностях.

  1. Вербальный интеллект

В начале 20 века психологи пришли к выводу, что существует прямая корреляция между объемом словарного запаса и силой, а также общим жизненным успехом.

Чем богаче словарный запас, а также их способность умно использовать его в различных ситуациях, тем более успешными и уверенными они будут в своей жизни в целом — в учебе, работе, социальной и личной жизни (Бузан, 2002).

Слова обладают невероятной силой, чем больше вы можете использовать силу слов, тем больше вы способны убеждать, мотивировать, увлекать и влиять на тех, кто вас окружает, и поэтому очевидно, почему компании захотят использовать вербальный интеллект оценки в качестве предиктора будущего успеха как часть их инструментов выбора и развития.

Как мы уже упоминали, это богатство словарного запаса, а также то, насколько умен человек в использовании этого словаря для передачи сообщения, и, соответственно, вербальные оценки обычно включают различные типы вопросов для измерения этого.

Вопросы включают синонимы, антонимы, аналогии и вербальное понимание.

а. Проверка синонима

Синоним — это слово, имеющее идентичное или эквивалентное значение другому слову того же языка.

Примеры синонимов: улучшать и улучшать, расслабляться и успокаивать, спрашивать и спрашивать.
В тесте на вербальное мышление кандидата можно попросить выбрать только одно слово из, например, пяти слов в скобках, которое наиболее близко по значению к слову, написанному заглавными буквами:

  • GLUTINOUS (грубый, липкий, голодный, яркий, пустынный)
  • ОСВЕЩЕНИЕ (настоящее, достоверное, информативное, рациональное, связное)
б. Тест на антоним

Напротив, антоним — это слово, значение которого противоположно другому слову того же языка.

Примеры антонимов: громкий и тихий, высокий и низкий, быстрый и медленный. В этих тестах проверяются ваши знания языка и способность быстро определять слова с противоположным значением.

В каждом из приведенных ниже вопросов теста «Антоним» кандидат должен выбрать только одно слово из пяти слов в скобках, которое наиболее противоположно по значению слову, написанному заглавными буквами.

  • RIBALD (благородный, привлекательный, серьезный, этичный, строгий)
  • TOUCHY (послушный, удачливый, гениальный, отзывчивый, стойкий)
c.Тест аналогии

Аналогию можно определить как подобие отношений, где необходимо обосновать ответ из параллельного случая. Вопрос в тесте по аналогии может быть в форме «A относится к B, как C относится?», Как в приведенном ниже примере:

  • ШЛЕМ предназначен для защиты, как TIARA: для украшения, королевы, волос, королевской власти. , голова
г. Понимание

Понимание — это еще один способ оценки вербальных способностей. Вопрос на понимание может поступать в формате ниже, в котором кандидата просят вернуть удаленное слово из абзаца.

В этом примере из абзаца удалено 15 различных слов, которые были перечислены в случайном порядке под ним, и для проверки вербальных навыков и способностей к пониманию кандидат должен правильно восстановить 15 слов.

Так же, как __________________ (1) __________________ (2) был на полпути через __________________ (3) его самый __________________ (4) и трудный __________________ (5), случилось то __________________ (6), что могло произойти, и черт возьми был выпущен как Бен __________________ (7) кот через столовую _______________ (8).Помимо того, что __________________ (9) почти лаял в доме и заглушил вторую половину вопроса, Бен __________________ (10) пересек столовую в __________________ (11) пятне, прежде чем швырнуть __________________ (12) в __________________ (13) дверь. с ______________ (14) __________________ (15).

Окно, вопрос, сам, тошнотворный, несчастный, вовлеченный, худший, видел, выстрел, авария, спрашивающий, интервьюер, полностью, коричневый, кухня.

2- Числовой интеллект

Каждому в жизни нужен некоторый уровень числовых способностей, будь то расчет еженедельного счета за покупки или планирование того, как лучше всего потратить свою ежемесячную зарплату.

Тесты на числовые способности разработаны для того, чтобы оценить, насколько хорошо вы умеете рассуждать с помощью чисел. Вопросы в рамках этих оценок могут сочетать как простые математические вычисления, так и задачи, требующие демонстрации методологии логического мышления.

Подмножество вопросов может быть использовано для оценки численного решения задач, и в то время как фактический математический расчет может быть довольно простым; тем не менее, кандидаты в первую очередь оцениваются по их способности использовать свои базовые знания чисел для быстрого и точного решения проблемы структурированным способом.

Компании предпочитают числовые оценки, потому что цифры являются международными, и, следовательно, нет опасений какого-либо неблагоприятного воздействия из-за культурных предубеждений, когда кто-то выиграет, если он / она происходит из определенной культуры, уменьшено, и, следовательно, можно найти их широко используемыми отдельно или вместе с тестами IQ.

Индивидуальные тесты включают числовые последовательности и ментальный арифметический тест, которые направлены на измерение способностей человека к математическим вычислениям, распознавания закономерностей и способности рассуждать с помощью чисел.

а. Проверка числовой последовательности

При оценке числовой последовательности кандидат должен определить закономерность, которая встречается в последовательности. Числа в последовательности могут увеличиваться, уменьшаться или, в некоторых случаях, может существовать последовательность / шаблон увеличения и уменьшения, когда ожидается, что кандидат определит и решит вопрос соответственно.

Обучая такие числовые ряды, вы быстрее поймете логику, лежащую в основе ряда, потому что некоторые из этих рядов используются довольно часто и потому что вы тренируетесь читать эти ряды.

В приведенных ниже примерах кандидата просят заполнить недостающие числа, указанные в вопросительном знаке, например:

  • 0, 1, 4, 9, 16, 25, 36, 49,?
  • 9, 18, 27,?, 45,?, 63
  • 100, 96,75, 93,5, 90,25, 87,?
г. Тест на ментальную арифметику

Кто-то может возразить, что ментальная арифметика не преподается в сегодняшней системе образования в той же степени, что и несколько лет назад, когда дети так хорошо знали таблицу умножения, что могли отвечать на такие вопросы, как 9, умноженное на 8 или 6 умножить на 7, даже не задумываясь, что понятно, учитывая распространенное использование калькуляторов, компьютеров, iPad; Сказав это, умственные арифметические способности — это бесценный талант, которым можно располагать, а также отличный способ тренировки мозга.

Ниже приведены примеры из теста скорости мысленной арифметики, во время которого тестируемый должен решать математические задачи с возрастающей сложностью по мере прохождения теста.

Они должны уметь решать эти проблемы быстро и спокойно, пытаясь найти лучший и эффективный способ решения вопросов.

  • Сколько 11 умножить на 13?
  • Что составляет 45% от 300?
  • Умножьте 8 на 15 и разделите результат на 6.
c.Работа с числами

Эти тесты разработаны для оценки способности кандидата работать с числами и мыслить численно. Приведенная ниже загадка представляет собой пример вопросов, включенных в тест числового мышления:

Если бы Алиса дала Сьюзен 6 долларов, каждая из них имела бы соотношение 2: 1; однако, если бы Сьюзен дала Алисе 1 доллар, соотношение было бы 1: 3. Сколько денег было у Алисы и Сьюзен до того, как они обменялись деньгами?

Вот ссылка на числовую оценку рассуждений Aon e-Cut, которую вы можете потренировать перед следующей цифровой оценкой:

http: // getstarted. cut-e.com/home/?modul=k%2bYgw4iavL%2f6JA%2bs94zDpg%3d%3d&theme=16peDn%2bPxEIKyLUqZjlX4Q%3d%3d

3- Технические возможности

Сегодняшний мир становится все более востребованным. ; оценка технических способностей становится крайне важной, учитывая, что люди с более сильными научными и техническими способностями, по прогнозам, будут иметь более высокий потенциал для освоения технологий по сравнению с людьми с более низкими техническими способностями.

Таким образом, отбор кандидатов, продемонстрировавших высокий уровень технических способностей в профессиях, зависящих от технологий, является значительно более экономичным с точки зрения как кривой обучения, так и общей производительности.

По мере того, как новые технологии продолжают возникать и развиваться, для работодателей важно, чтобы в их распоряжении были объективные способы распознавания кандидатов, которые способны быстро изучить эти новые технологии и могут применять их в своей работе для решения сложных задач. вопросы.

Ниже приведены два примера из теста, который был разработан для измерения общих научных знаний, технических способностей и силы механического мышления и логики.

  • На какой процент видимая фигура закрашена?

  • Какова площадь в квадратных единицах на рисунке ниже?

4- Логическое рассуждение

«Логика», первоначально означающая «слово», теперь используется для обозначения «мышления» и «рассуждения», при этом логическим индивидуумом считается тот, кто имеет навыки рассуждения или использования разума структурированным и убедительным образом.

Оценка логического мышления оценивает степень, в которой кандидаты демонстрируют аналитическое мышление и могут применять здравый процесс рассуждения для решения проблем. Есть 2 типа вопросов логического рассуждения, с которыми может столкнуться кандидат; вопросы чистой логики и прогрессивных матриц.

а. Чистая логика

Это задание состоит из вопросов разного объема и разных уровней сложности.

В отличие от тестов на технические навыки, логическое рассуждение не требует от кандидата специальных знаний математики или словарного запаса, чтобы уметь решать вопросы теста, только способность мыслить ясно и аналитически.

Ниже приведен вопрос из оценки логического рассуждения, которая оценивает чистую логику.

  • Какой номер должен заменить вопросительный знак?
б. Тест с прогрессивными матрицами

Тесты с прогрессивными матрицами были разработаны для оценки и проверки способности кандидата распознавать шаблоны и конструкции, а также его способности мыслить логически, при этом оставаясь открытым для изучения различных потенциальных вариантов, которые также могут привести к правильному ответу .

В этих тестах, как вы увидите в следующих примерах, кандидатам представлен узор в виде массива квадратов, где один из квадратов был удален, и им необходимо выбрать правильный недостающий квадрат из нескольких вариантов.

Поэтому важно взглянуть на матрицу, чтобы узнать, какая закономерность возникает, просматривая каждую строку и каждый столбец, просматривая массив в целом или глядя на связь между различными квадратами внутри массива.

Как правило, эти тесты начинаются с простых шаблонов (массивы 2X2) и начинают усложняться по мере того, как кандидат продвигается в своем тесте (массивы 4X4). Опять же, при прохождении этих тестов важно сохранять непредвзятость, чтобы иметь возможность думать обо всех возможных вариантах паттернов и последовательностей.
Ниже приведены 2 вопроса о массивах 2X2 и 4X4.

  • Какой квадрат отсутствует?

  • Какой раздел отсутствует?

Технологии в оценках

Технологии изменили ландшафт большинства отраслей и дисциплин, и оценка талантов не исключение.

Работодатели теперь попросят вас пройти большинство экзаменов онлайн, а с помощью технологий компьютерного адаптивного тестирования (CAT) вам будет представлен динамический выбор вопросов из большого набора элементов в зависимости от того, как вы отвечаете, например, если вы правильно ответили на первый вопрос, следующий вопрос будет более сложным, но если вы не ответили правильно, следующий вопрос будет менее сложным.

Сейчас очень часто просят заполнить Gamified версию ваших оценок, что становится все более популярным.Чтобы получить представление о том, как работают геймифицированные оценки, вы можете загрузить приложение Pymetrics и поиграть в некоторые из их игр.

Практика тестирования когнитивных способностей

Практика оценки когнитивных способностей — это то, что вы всегда должны учитывать.

Пройдите как можно больше тестов, постарайтесь выяснить, какие компоненты наиболее сложны для вас, и отработайте эти компоненты больше, чем другие.

Речь идет не только о решении проблем, но и о поиске истинного обоснования решений, и, соответственно, речь идет не о том, чтобы знать наизусть множество заданий и решений, а о понимании вопросов, чтобы вы могли больше легко ответить на подобные вопросы в будущем.

Нужно больше практики? Попробуйте практические тесты от JobTestPrep. Чтобы узнать больше, нажмите здесь.

Ссылки
  • Buzan, T. (2002). Сила вербального интеллекта . HarperCollins Publishers
  • Кэмпбелл, Дж. П. (1990a). Моделирование задачи прогнозирования производительности в промышленной и организационной психологии. В M. D. Dunnette & L. M. Hough (Eds.), Справочник по промышленной и организационной психологии (том 1, стр. 687-732). Пало-Альто, Калифорния: Консультации психологов Press.
  • Готтфредсон, Л. С. (1997a). Основная наука об интеллекте: редакционная статья с 52 подписчиками, историей и библиографией. Разведка , 24, 13-23.
  • Судья Т. А., Колберт А. Э. и Илиес Р. (2004). Интеллект и лидерство: количественный обзор и проверка теоретических положений. Журнал прикладной психологии , 89, 542-552.
  • Кунсел, Н. Р., Хезлетт, С. А., и Онес, Д. С. (2004). Академическая успеваемость, карьерный потенциал, творческий потенциал и эффективность работы: можно ли построить все это? Журнал личности и социальной психологии , 86, 148-161.

Добавить комментарий

Ваш адрес email не будет опубликован. Обязательные поля помечены *